Buy BOOKS at Discounted Price

Probability

Class 12th Mathematics RD Sharma Volume 2 Solution
Exercise 31.1
  1. Ten cards numbered 1 through 10 are placed in a box, mixed up thoroughly and…
  2. the youngest is a girl, Assume that each child born is equally likely to be a…
  3. at least one is a girl? Assume that each child born is equally likely to be a…
  4. Given that the two numbers appearing on throwing two dice are different. Find…
  5. A coin is tossed three times, if head occurs on first two tosses, find the…
  6. A die is thrown three times, find the probability that 4 appears on the third…
  7. Compute P (A/B), if P (B) = 0.5 and P (A∩B) = 0.32
  8. If P (A) = 0.4, P (B) = 0.3 and P(B/A)=0.5, find P(A∩B) and P(A/B).…
  9. If A and B are two events such that p (a) = 1/3 , p (b) = 1/5 and p (a union b)…
  10. A couple has two children. Find the probability that both the children are (i)…
Exercise 31.2
  1. From a pack of 52 cards, two are drawn one without replacement. Find the…
  2. From a pack of 52 cards, 4 are drawn one by one without replacement. Find the…
  3. Find the chance of drawing 2 white balls in succession from a bag containing 5…
  4. A bag contains 25 tickets, numbered from 1 to 25. A ticket is drawn and then…
  5. From a deck of cards, three cards are drawn on by one without replacement. Find…
  6. both are kings Two cards are drawn without replacement from a pack of 52…
  7. the first is a king and the second is an ace Two cards are drawn without…
  8. the first is a heart and second is red. Two cards are drawn without…
  9. A bag contains 20 tickets, numbered from 1 to 20. Two tickets are drawn without…
  10. An urn contains 3 white, 4 red and 5 black balls. Two balls are drawn one by…
  11. A bag contains 5 white, 7 red and 3 black balls. If three balls are drawn one…
  12. A card is drawn from a well-shuffled deck of 52 cards and then a second card…
  13. An urn contains 10 black and 5 white balls. Two balls are drawn from the urn…
  14. Three cards are drawn successively, without replacement from a pack of 52 well…
  15. A box of oranges is inspected by examining three randomly selected oranges…
  16. A bag contains 4 white, 7 black 5 red balls. Three balls are drawn one after…
Exercise 31.3
  1. If P (A) = 7/13, P (B) =9/13 and P (A ∩ B) = 4/13, find p (A/B).
  2. If A and B are events such that P (A) =0.6, P (B) = 0.3 and P (A ∩ B) = 0.2,…
  3. If A and B are two events such that P (A ∩ B) = 0.32 and P (B) = 0.5, find P…
  4. If P (A) = 0.4, P (B) = 0.8, P (B/A) = 0.6. Find P (A/B) and P (A ∪ B).…
  5. P (A) = 1/3, P (B)=1/4 and P(A ∪ B) = 5/12, find P (A/B) and P (B/A). If A and…
  6. P (A) = 6/11, P (B) = 5/11 and P (A ∪ B) = 7/11, find P (A ∩ B), P (A/B), P…
  7. P (A)= 7/13, P (B)=9/13 and P(A ∩ B)=4/13, find P (A/B). If A and B are two…
  8. P(A)=1/2,P (B)=1/3 and P(A ∩ B) = 1/4, find P (A/B), P(B/A), P (A/B) and P…
  9. If A and B are two events such that 2P (A) = P (B) = 5/13 and P (A/B) = 2/5,…
  10. If P (A)=6/11,P (B)=5/11 and P (A ∪ B) = 7/11, find i. P (A ∩ B) ii. P (A /B)…
  11. A=Heads on third toss, B=Heads on first two tosses A coin is tossed three…
  12. A=At least two heads, B=At most two heads A coin is tossed three times. Find P…
  13. A=At most two tails, B=At least one tail. A coin is tossed three times. Find P…
  14. A=Tail appears on one coin, B= One coin shows head. Two coins are tossed once.…
  15. A=No tail appears, B=No head appears. Two coins are tossed once. Find P(A/B)…
  16. A die is thrown three times. Find P(A/B) and P (B/A), if A=4 appears on the…
  17. Mother, father and son line up at random for a family picture. If A and B are…
  18. A dice is thrown twice, and the sum of the numbers appearing is observed to be…
  19. Two dice are thrown. Find the probability that the numbers appeared has the…
  20. A pair of dice is thrown. Find the probability of getting 7 as the sum, if it…
  21. A pair of dice is thrown. Find the probability of getting 7 as the sum if it…
  22. A die is rolled. If the outcome is an odd number, what is the probability that…
  23. A pair of dice is thrown. Find the probability of getting the sum 8 or more,…
  24. Find the probability that the sum of the numbers showing on two dice is 8,…
  25. Two numbers are selected at random from integers 1 through 9 If the sum is…
  26. A die is thrown twice, and the sum of the numbers appearing is observed to be…
  27. Two dice are thrownand it is known that the first die shows a 6 Find the…
  28. A pair of dice is thrown. Let E be the event that the sum is greater than or…
  29. The probability that a student selected at random from a class will pass in…
  30. The probability that a certain person will buy a shirt is 0.2, the probability…
  31. In a school, there are 1000 students, out of which 430 are girls. It is known…
  32. Ten cards numbered 1 through 10 are placed in a box, mixed up thoroughly and…
  33. Assume that each born child is equally likely to be a boy or a girl. If a…
Exercise 31.4
  1. A coin is tossed thrice and all the eight outcomes are assumed equally likely.…
  2. Prove that is throwing a pair of dice, the occurrence of the number 4 on the…
  3. A card is drawn form a pack of 52 cards so that each card is equally likely to…
  4. A coin is tossed three times. Let the events A,B and C be defined as follows:…
  5. If A and B be two events such that P(A) = 1/4, P (B) = 1/3 and P (A ∪ B) = 1/2,…
  6. Given two independent events A and B such that P (A)=0.3 and P(B) = 0.6 Find i.…
  7. If P (not B) = 0.65, P (A ∪ B) = 0.85, and A and B are independent events, then…
  8. If A and B are two independent events such that p (bar a integrate erdection b)…
  9. A and B are two independent events. The probability that A and B occur is 1/6…
  10. If A and B are two independent events such that P (A ∪ B) = 0.60 and P(A)=…
  11. A die Is tossed twice. Find the probability of getting a number greater than 3…
  12. Given the probability that A can solve a problem is 2/3 and the probability…
  13. An unbiased die is tossed twice. Find the probability of getting 4,5, or 6 on…
  14. A bag contains 3 red and 2 black balls. One ball Is drawn from it at random.…
  15. Three cards are drawn with replacement from a well shuffled pack of cards.…
  16. An article manufactured by a company consists of two parts X and Y. In the…
  17. The probability that A hits a target is 1/3 and the probability that B hits…
  18. An anti-aircraft gun can take a maximum of 4 shots at an enemy plane moving…
  19. The odds against a certain event are 5 to 2 and the odds in favour of another…
  20. A die is thrown thrice. Find the probability of getting an odd number at least…
  21. Two balls are drawn at random with replacement from a box containing 10 black…
  22. An urn contains 4 red and 7 black balls. Two balls are drawn at random with…
  23. The probabilities of two students A and B coming to the school in time are 3/7…
  24. Two dice are thrown together and the total score is noted. The event E, F and…
  25. Let A and B be two independent events such that P(A) = p1 and P(B) = p2.…
Exercise 31.5
  1. A bag contains 6 black and 3 white balls. Another bag contains 5 black and 4…
  2. A bag contains 3 red and 5 black balls and a second bag contains 6 red and 4…
  3. Two balls are drawn at random with replacement from a box containing 10 black…
  4. Two cards are drawn successively without replacement from a well-shuffled deck…
  5. A speaks truth in 75% and B in 80% of the cases. In what percentage of cases…
  6. Kamal and Monica appeared for an interview for two vacancies. The probability…
  7. A bag contains 3 white, 4 red and 5 black balls. Two balls are drawn one after…
  8. A bag contains 8 red and 6 green balls. Three balls are drawn one after another…
  9. Arun and Tarun appeared for an interview for two vacancies. The probability of…
  10. A and B toss a coin alternately till one of them gets a head and wins the…
  11. Two cards are drawn from a well shuffled pack of 52 cards, one after another…
  12. Tickets are numbered from 1 to 10. Two tickets are drawn one after the other…
  13. In a family, the husband tells a lie in 30% cases and the wife in 35% cases.…
  14. A husband and wife appear in an interview for two vacancies for the same post.…
  15. A bag contains 7 white, 5 black and 4 red balls. Four balls are drawn without…
  16. A, B, and C are independent witness of an event which is known to have…
  17. A bag contains 4 white balls and 2 black balls. Another contains 3 white balls…
  18. A bag contains 4 white, 7 black and 5 red balls. 4 balls are drawn with…
  19. Three cards are drawn with replacement from a well shuffled pack of 52 cards.…
  20. A bag contains 4 red and 5 black balls, a second bag contains 3 red and 7…
  21. A can hit a target 3 times in 6 shots, B:2 times in 6 shots and C:4 times in 4…
  22. The probability of student A passing an examination is 2/9 and of student B…
  23. There are three urns A, B and C. Urn A contains 4 red balls and 3 black balls.…
  24. X is taking up subjects - Mathematics, Physics and Chemistry in the…
  25. A and B take turns in throwing two dice, the first to throw 9 being awarded…
  26. A, B and C in order toss a coin. The one to throw a head wins. What are their…
  27. Three persons A, B, C throw a die in succession till one gets a ‘six’ and wins…
  28. A and B take turns in throwing two dice, the first to throw 10 being awarded…
  29. There are 3 red and 5 black balls in bag ‘A’; and 2 red and 3 black balls in…
  30. Fatima and John appear in an interview for two vacancies for the same post.…
  31. A bag contains 8 marbles of which 3 are blue and 5 are red. One marble is…
  32. An urn contains 7 red and 4 blue balls. Two balls are drawn at random with…
  33. A card is drawn from a deck of 52 cards. The outcome is noted, the card is…
  34. Out of 100 students, two sections of 40 and 60 are formed. If you and your…
  35. In a hockey match, both teams A and B scored same number of goals up to the…
  36. A and B throw a pair of die alternately. A wins the game if he gets a total of…
Exercise 31.6
  1. A bag A contains 5 white and 6 black balls. Another bag B contains 4 white and…
  2. A purse contains 2 silver and 4 copper coins. A second purse contains 4 silver…
  3. One bag contains 4 yellow and 5 red balls. Another bag contains 6 yellow and 3…
  4. A bag contains 3 white and 2 black balls, and another bag contains 2 white and…
  5. The contents of three bags I, II and III are as follows: Bag I: 1 white, 2…
  6. An unbiased coin is tossed. If the result is a head, a pair of unbiased dice is…
  7. A factory has two machines A and B. Past records show that the machine A…
  8. The bag A contains 8 white and 7 black balls while the bag B contains 5 white…
  9. A bag contains 4 white and 5 black balls and another bag contains 3 white and 4…
  10. One bag contains 4 white and 5 black balls. Another bag contains 6 white and 7…
  11. An urn contains 10 white and 3 black balls. Another urn contains 3 white and 5…
  12. A bag contains 6 red and 8 black balls and another bag contains 8 red and 6…
  13. Three machines E1,E2,E3 in a certain factory produce 50%, 25% and 25%…
Exercise 31.7
  1. The contents of the urns I,II,III are as follows: Urn I: 1 white, 2 black and 3…
  2. A bag A contains 2 white and 3 red balls and a bag B contains 4 white and a bag…
  3. Three urns contains 2 white and 3 black balls; 3 white and 2 black balls; 4…
  4. The contents of the three urns are as follows: Urn I contains 7 white and 3…
  5. Suppose a girl thrown a die. If she gets 1 or 2, she tosses a coin three times…
  6. Two groups are competing for the positions of the Board of Directors of…
  7. Suppose 5 men out of 100 and 25 women out of 1000 are good orators. An orator…
  8. A letter is known to have come either from LONDON or CLIFTON. On the envelope…
  9. In a class 5% of the boys and 10% of the girls have an IQ of more than 150. In…
  10. A factory has three machines X,Y and Z producing 1000, 2000 and 3000 bolts per…
  11. An insurance company insured 3000 scooters, 4000 cars and 5000 trucks. The…
  12. Suppose we have four boxes A,B,C,D containing coloured marbles as given below:…
  13. A manufacturer has three machine operators A,B and C. The first operator A…
  14. An item is manufactured by three machines A,B and C. Out of the total number…
  15. There are three coins. One is two - headed coin (having head on both faces),…
  16. In a factory, machine A produces 30% of the total output, machine B produces…
  17. A company has two plants to manufacture bicycles. The first plant manufactures…
  18. Three urns A,B and C contain 6 red and 4 white; 2 red and 6 white; and 1 red…
  19. In a group of 400 people, 160 are smokers and non - vegetarian, 100 are…
  20. A factory has three machines A,B and C, which produce 100,200 and 300 items of…
  21. A bag contains 1 white and 6 red balls, and a second bag contains 4 white and…
  22. In a certain college, 4% of boys and 1% of girls are taller than 1.75 metres.…
  23. For A,B and C the chances of being selected as the manager of a firm are in…
  24. Three persons A,B and C apply for a job of Manager in a private company.…
  25. An insurance company insured 2000 scooters and 3000 motorcycles. The…
  26. Of the students in a college, it is known that 60% reside in a hostel and 40%…
  27. There are three coins. One is two headed coin, another is a biased coin that…
  28. Assume that the chances of a patient having a heart attack is 40%. It is also…
  29. Coloured balls are distributed in four boxes as shown in the following table:…
  30. If a machine is correctly set up it produces 90% acceptable items. If it is…
  31. Bag A contains 3 red and 5 black balls, white bag B contains 4 red and 4 black…
  32. By examining the chest X - ray, probability that T.B is detected when a person…
  33. A test for detection of a particular disease is not fool proof. The test will…
  34. Let d1, d2, d3 be three mutually exclusive diseases. Let S be the set of…
  35. A is known to speak truth 3 times out of 5 times. He throws a die and reports…
  36. A speaks the truth 8 times out of 10 times. A die is tossed. He reports that…
  37. In answering a question on a multiple choice test, a student either knows the…
  38. A laboratory blood test is 99% effective in detecting a certain disease when…
  39. There are three categories of students in a class of 60 students: A: Very…
Very Short Answer
  1. A four digit number is formed using the digits 1, 2, 3, 5 with no repetitions. Write…
  2. When three dice are thrown, write the probability of getting 4 or 5 on each of the dice…
  3. Three digit numbers are formed with the digits 0, 2, 4, 6 and 8. Write the probability of…
  4. A ordinary cube has four plane faces, one face marked 2 and another face marked 3, find…
  5. Three numbers are chosen from 1 to 20. Find the probability that they are consecutive.…
  6. 6 boys and 6 girls sit in a row at random. Find the probability that all the girls sit…
  7. If A and B are two independent events such that P(A) = 0.3 and . Find P(B).…
  8. An unbiased die with face marked 1, 2, 3, 4, 5, 6 is rolled four times. Out of 4 face…
  9. If A and B are two events write the expression for the probability of occurrence of…
  10. Write the probability that a number selected at random from the set of first 100…
  11. In a competition A, B and C are participating. The probability that A wins is twice…
  12. If A, B, C are mutually exclusive and exhaustive events associated to a random…
  13. If two events A and B are such that , and , find
  14. If A and B are two independent events, then write ) in terms of P(A) and P(B).…
  15. If P(A) = 0.3, P(B) = 0.6, P(B/A) = 0.5, find
  16. If A, B and C are independent events such that P(A) = P(B) = P(C) = p, then find the…
  17. If A and B are independent events then write expression for P (exactly one of A, B…
  18. If A and B are independent events such that P(A) = p, p(B) = 2p and P (Exactly one of…
Mcq
  1. If one ball is drawn at random from each of three boxes containing 3 white and 1 black,…
  2. A and B draw two cards each, one after another, from a pack of well-shuffled pack of 52…
  3. A and B are two events such that P(A) = 0.25 and P(B) = 0.50. The probability of both…
  4. The probabilities of a student getting I, II and III division in an examination are…
  5. India play two matches each with West Indies and Australia. In any match the…
  6. Three faces of an ordinary dice are yellow, two faces are red and one face is blue. The…
  7. The probability that a leap year will have 53 Friday 53 Saturdays is Mark the correct…
  8. A person writes 4 letters and addresses 4 envelopes. If the letters are placed in the…
  9. A speaks truth in 75% cases and B speaks truth in 80% cases. Probability that they…
  10. Three integers are chosen at random from the first 20 integers. The probability that…
  11. Out of 30 consecutive integers, 2 are chosen at random. The probability that their sum…
  12. A bag contains 5 black balls, 4 white balls and 3 red balls. If a ball is selected…
  13. Two dice are thrown simultaneously. The probability of getting a pair of aces is Mark…
  14. An urn contains 9 balls two of which are red, three blue and four black. Three balls…
  15. A coin is tossed three times. If events A and B are defined as A = Two heads come, B =…
  16. Five persons entered the lift cabin on the ground floor of an 8 floor house. Suppose…
  17. A box contains 10 good articles and 6 with defects. One item is drawn at random. The…
  18. A box contains 6 nails and 10 nuts. Half of the nails and half of the nuts are rusted.…
  19. A bag contains 5 brown and 4 white socks. A man pulls out two socks. The probability…
  20. If S is the sample space and p (a) = {1}/{3} p (b) and S = A∪B, where A and B are…
  21. If A and B are two events, then p ( bar {a} integrate erdection b ) = Mark the…
  22. If P (A ∪ B) = 0.8 and P (A ∩ B) = 0.3, then p ( bar {a} ) + p ( overline{b} ) = Mark…
  23. A bag X contains 2 white and 3 black balls and another bag Y contains 4 white and 2…
  24. Two persons A and B take turns in throwing a pair of dice. The first person to throw 9…
  25. The probability that in a year of 22nd century chosen at random, there will be 53…
  26. From a set of 100 cards numbered 1 to 100, one card is drawn at random. The…
  27. If A and B are two events such that p (a) = {4}/{5} , p ( a integrate erdection b )…
  28. If A and B are two events associated to a random experiment such that p ( a integrate…
  29. Associated to a random experiment two events A and B are such that P(B) = 3/5, P(A/B)…
  30. If P(A) = 3/10, P(B) = 2/5 and P(A ∪B) = 3/5, then P(A/B) + P(B/A) equals Mark the…
  31. Let P(A) = 7/13, P(B) = 9/13 and P(A ∩ B) = 4/13. Then p ( bar {a}/b ) = Mark the…
  32. If p (a) = {2}/{5} , p (b) = frac {3}/{10} and p ( a integrate erdection b ) =…
  33. If A and B are two events such that p (a) = {3}/{8} , p (b) = frac {1}/{3} , p…
  34. Let A and B be two events such that p (a) = {3}/{8} , p (b) = frac {5}/{8} and p…
  35. If p (b) = {3}/{5} , p (a/b) = frac {1}/{2} and p ( a union b ) = {4}/{5} then…
  36. If P(A) = 0.4, P(B) = 0.8 and P(B/A) = 0.6, then P(A ∪ B) = Mark the correct…
  37. If p (b) = {3}/{5} , p (a/b) = frac {1}/{2} and p ( a union b ) = {4}/{5} then…
  38. If A and B are two events such that P(A) = 0.4, P(B) = 0.3 and P(A ∪ B) = 0.5, then p…
  39. If A and B are two events such that A ≠ ϕ, B = ϕ, then Mark the correct alternative in…
  40. If A and B are two events such that P(A) ≠ 0 and P(B) ≠ 1, then p ( bar {a} /…
  41. If the events A and B are independent, then P(A ∩ B) is equal to Mark the correct…
  42. If A and B are two independent events with p (a) = {3}/{5} andp (b) = frac {4}/{9}…
  43. If A and B are two independent events such that P(A) = 0.3, P(A ∪ B) = 0.5, then…
  44. A flash light has 8 batteries out of which 3 are dead. If two batteries are selected…
  45. A bag contains 5 red and 3 blue balls. If 3 balls are selected drawn at random without…
  46. A bag contains 5 red and 3 blue balls. If 3 balls are drawn at random without…
  47. In a college 30% students fail in Physics, 25% fail in Mathematics and 10% fail in…
  48. Three person, A, B and C fire a target in turn starting with A. Their probabilities of…
  49. A and B are two students. Their chances of solving a problem correctly are 1/3 and 1/4…
  50. Two cards are drawn from a well shuffled deck of 52 playing cards with replacement.…
  51. A box contains 3 orange balls, 3 green balls and 2 blue balls. Three balls are drawn…
  52. If two events are independent, then Mark the correct alternative in each of the…
  53. Two dice are thrown. If it is known that the sum of the numbers on the dice was less…
  54. If A and B are such that p ( a union b ) = {5}/{9} and p ( bar {a} union…
  55. If A and B are two events such that P(A/B) = p, P(A) = p, P(B) = 1/3 and p ( a union…
  56. A die is thrown and a card is selected at random from a deck of 52 playing cards. The…
  57. Assume that in a family, each child is equally likely to be a boy or a girl. A family…
  58. Let A and B be two events. If P(A) = 0.2, P(B) = 0.4, P(A ∪ B) = 0.6, then P(A/B) is…
  59. Let A and B be two events such that P(A) = 0.6, P(B) = 0.2 and P(A/B) = 0.5. Then p (…

Exercise 31.1
Question 1.

Ten cards numbered 1 through 10 are placed in a box, mixed up thoroughly and then one card is drawn randomly. If it is known that the number on the drawn card is more than 3, what is the probability that it is an even number?


Answer:

Ten cards are numbered, hence the sample space is


S = {1, 2, 3, 4, 5, 6, 7, 8, 9, 10}, n(S) = 10


Let event A be the even number card drawn, then


A = {2, 4, 6, 8, 10}, n(A)=5


Now probability that an even card is drawn is



Let event B be the cards greater than 3, then


B = {4, 5, 6, 7, 8, 9, 10}, n(B) = 7



So the probability of A∩B


So, A∩B = {4, 6, 8, 10}, n(A∩B)=4



So the required probability will become



Hence the probability that an even card is drawn out of cards that are more than 3 is



Question 2.

Assume that each child born is equally likely to be a boy or a girl. If a family has two children, what is the conditional probability that both are girls given that

the youngest is a girl,


Answer:

Let b and g represents the boy and the girl child respectively.


Now if a family has two children, the sample space will be


S = {(b, b), (b, g), (g, b), (g, g)}, n(S)=4


Let A be the event that both children are girls, the


A= {(g, g)}, n(A)=1


So the probability that both children are girls is



If the youngest is a girl


Let B be the event that the youngest child is a girl


Then B = {(b, g), (g, g)}, n(B) = 2


And the corresponding probability becomes



The sample space for the both girls is girl child and the youngest is a girl will become


(A∩B) = {(g, g)}, n(A∩B)=1


And the corresponding probability becomes



So the conditional probability that both are girls given that the youngest is a girl is




Question 3.

Assume that each child born is equally likely to be a boy or a girl. If a family has two children, what is the conditional probability that both are girls given that

at least one is a girl?


Answer:

At least one is a girl


Let C be the event that at least one is a girl


Then C = {(b, g), (g, b), (g, g)}, n(C) = 3


And the corresponding probability becomes



The sample space for the both girls is girl child and at least one is a girl will become


(A∩C) = {(g, g)}, n(A∩C)=1


And the corresponding probability becomes



So the conditional probability that both are girls given that the youngest is a girl is




Question 4.

Given that the two numbers appearing on throwing two dice are different. Find the probability of the event ‘the sum of numbers on the dice is 4’.


Answer:

Let S be the sample space of throwing two dice and the two numbers are different, then the sample space is


S = {(1, 2), (1, 3), (1, 4), (1, 5), (1, 6)


(2, 1), (2, 3), (2, 4), (2, 5), (2, 6)


(3, 1), (3, 2), (3, 4), (3, 5), (3, 6)


(4, 1), (4, 2), (4, 3), (4, 5), (4, 6)


(5, 1), (5, 2), (5, 3), (5, 4), (5, 6)


(6, 1), (6, 2), (6, 3), (6, 4), (6, 5)}


n(S) = 30


Let B be the event that the sum if the numbers on the dice is 4, then the sample space becomes


B = {(1, 3), (3, 1)}, n(B) = 2


So the probability of the event ‘the sum of numbers on the dice is 4’ is




Question 5.

A coin is tossed three times, if head occurs on first two tosses, find the probability of getting head on third toss.


Answer:

If h and t represents heads and tails respectively, then when a coin is tossed three times, the sample space becomes


S = {(HHH, HHT, HTH, HTT, THH, THT, TTH, TTT)},n(S)=8


Let E be the event of head occurring on the third toss, then the favorable outcomes will be


E = {(HHH, HTH, THH, TTH)}, n(E) = 4


So the corresponding probability will be



Let F be the event that head occurs on first two toss, then the favorable outcomes will be


F = {(HHH,HHT)}, n(F) = 2


So the corresponding probability will be



And the favorable outcome for getting head on all the three toss will be


(E∩F)={(HHH)}, n(E∩F)=1


And the corresponding probability becomes



So if head occurs on first two tosses, the probability of getting head on third tosses




Question 6.

A die is thrown three times, find the probability that 4 appears on the third toss if it is given that 6 and 5 appear respectively on first two tosses.


Answer:

When a die is thrown three times the number of all favorable outcomes are n(S) = 216


Let E be the event that 4 occurs on the third toss, then the favorable outcomes will be


E ={(1, 1, 4),(1, 2, 4), (1, 3, 4), (1, 4, 4), (1, 5, 4), (1, 6, 4)


(2, 1, 4),(2, 2, 4), (2, 3,4), (2, 4, 4), (2, 5, 4), (2, 6, 4)


(3, 1, 4), (3, 2, 4), (3, 3, 4), (3, 4, 4), (3, 5, 4), (3, 6,4)


(4, 1 ,4), (4, 2, 4), (4, 3, 4), (4, 4, 4), (4, 5, 4), (4, 6, 4)


(5, 1, 4), (5, 2,4), (5, 3, 4),(5,4, 4), (5, 5, 4), (5, 6, 4)


(6, 1, 4),(6, 2, 4), (6, 3, 4), (6,4, 4),(6, 5, 4), (6, 6, 4)}


n(E) = 36


So the probability that 4 appears on the third toss is



Let F be the event that 6 and 5 appear on first two tosses respectively, and then the favorable outcomes will be


F = {(6, 5, 1), (6, 5, 2), (6, 5, 3), (6, 5, 4), (6, 5, 6)}


n(F) = 6


So the probability that 6 and 5 appear on first two tosses respectively will be



So the favorable outcomes that 6 and 5 appear respectively on first two tosses and 4 appears on the third toss will be


(E∩F) = {(6, 5, 4)}, n(E∩F) = 1


And the corresponding probability will be



So the conditional probability that if it is given that 6 and 5 appear respectively on first two tosses and 4 appears on the third toss will be




Question 7.

Compute P (A/B), if P (B) = 0.5 and P (A∩B) = 0.32


Answer:

Given: P(B) = 0.5, P(A∩B) = 0.32


To find: P(A|B)


We know that,



By substituting the values we get




is the required value



Question 8.

If P (A) = 0.4, P (B) = 0.3 and P(B/A)=0.5, find P(A∩B) and P(A/B).


Answer:

Given: P(A) = 0.4, P(B)=0.3, P(B|A) = 0.5


To find: P(A|B) and P(A∩B)


We know that,



By substituting the values we get



is the required value


Similarly,



By substituting the values we get



is the required value



Question 9.

If A and B are two events such that and , find P (A/B) and P (B/A).


Answer:

Given:


To find: P(A|B) and P(B|A)


We know that,



By substituting the values we get





We also know that,



By substituting the values we get



is the required value


Similarly,



By substituting the values we get



is the required valu



Question 10.

A couple has two children. Find the probability that both the children are

(i) males, if it is known that at least one of the children is male.

(ii) females, if it is known that the elder child is a female.


Answer:

Let b and g represents the boy and the girl child respectively.


Now if a family has two children, the sample space will be


S = {(b, b), (b, g), (g, b), (g, g)}, n(S)=4


(i) Let A be the event that both children are males, then


A= {(b, b)}, n(A)=1


So the probability that both children are males is



Let B be the event that at least one of the children is male


Then B = {(b, b), (b, g), (g, b)}, n(B) = 3


And the corresponding probability becomes



The sample space for the at least male and both being male will become


(A∩B) = {(b, b)}, n(A∩B)=1


And the corresponding probability becomes



So the conditional probability that both are males given that the at least one is male is



(ii) Let P be the event that both children are females, then


P= {(g, g)}, n(P)=1


So the probability that both children are females is



Let Q be the event that the elder child is a female


Then Q = {(g, b), (g, g)}, n(Q) = 2


And the corresponding probability becomes



The sample space for the elder child being female and both child being female will become


(P∩Q) = {(g, g)}, n(P∩Q)=1


And the corresponding probability becomes



So the conditional probability that if the elder child is female then both children are females is





Exercise 31.2
Question 1.

From a pack of 52 cards, two are drawn one without replacement. Find the probability that both of them are kings.


Answer:

Total number of all favorable cases is n(S) = 52


Let A be the event that first card drawn is a king. There are four kings in the pack. Hence, the probability of the first card is a king is



Let B be the event that second card is also king without replacement. Then there are 3 kings left in the pack as the cards are not replaced. Therefore, the probability of the second card is also king is



Then the probability of getting two kings without replacement is





The probability that both of them are kings is



Question 2.

From a pack of 52 cards, 4 are drawn one by one without replacement. Find the probability that all are aces (or, kings).


Answer:

Total number of all favorable cases is n(S) = 52


Let A be the event that first card drawn is ace (or, kings). There are four aces (or, kings) in the pack. Hence, the probability of the first card is ace (or, kings) is



Let B be the event that second card is also ace (or, kings) without replacement. Then there are 3 aces (or, kings) left in the pack as the cards are not replaced. Therefore, the probability of the second card is also ace (or, kings) is



Let C be the event that third card is also ace (or, kings) without replacement. Then there are 2 aces (or, kings) left in the pack as the cards are not replaced. Therefore, the probability of the third card is also ace (or, kings) is



Let D be the event that fourth card is also ace (or, kings) without replacement. Then there are 1 ace (or, kings) left in the pack as the cards are not replaced. Therefore, the probability of the fourth card is also ace (or, kings) is



Then the probability all are aces (or, kings) is




The probability that all are aces (or, kings) is



Question 3.

Find the chance of drawing 2 white balls in succession from a bag containing 5 red and 7 white balls, the ball first drawn not being replaced.


Answer:

Bag contains 5 red balls and 7 white balls. So the total number of all favorable cases is n(S) = 5+7=12


Let A represents first ball as white ball, and B be second ball as white ball.


Then the probability of drawing two white balls without replacement is


P(2 white balls without replacement)



(as there are 7 white balls in first draw out of 12 balls, and 6 white balls in second draw out of 11 balls as the balls are not replaced)


Hence the required probability is



Question 4.

A bag contains 25 tickets, numbered from 1 to 25. A ticket is drawn and then another ticket is drawn without replacement. Find the probability that both tickets will show even numbers.


Answer:

There are 25 tickets numbered from 1 to 25, so the sample space is


S = {1, 2, 3, …., 25}, n(S) = 25


Number of even numbered tickets from 1 to 25 is


{2, 4, 6, 8, 10, 12, 14, 16, 18, 20, 22, 24} = 12


Let A represents first ticket with even number and B represents second ticket with even number


Then the probability of both tickets being even number without replacement is


P(both tickets showing even number without replacement)



(as there are 12 even numbered tickets out of 25 tickets in first draw, and 11 even numbered tickets out of 24 tickets in second draw as the tickets are not replaced)


Hence the required probability is



Question 5.

From a deck of cards, three cards are drawn on by one without replacement. Find the probability that each time it is a card of spade.


Answer:

Total number of all favorable cases is n(S) = 52


Let A be the event that first card drawn is card of spade. There are 13 spade cards in the pack. Hence, the probability of the first card is spade is



Let B be the event that second card is also card of spade without replacement. Then there are 12 spade cards left in the pack as the cards are not replaced. Therefore, the probability of the second card is also spade card is



Let C be the event that third card is also spade card without replacement. Then there are 11 spade cards left in the pack as the cards are not replaced. Therefore, the probability of the third card is also a spade card is



Then the probability all three are spade cards without replacement is



(as there are 13 spade cards in the pack of 52 in first draw, 12 spade cards in the pack of 51cards in the second draw as the cards are not replaced and 11spadecards in the pack of 50 cards in the third draw as the cards are not replaced.)


The required probability is



Question 6.

Two cards are drawn without replacement from a pack of 52 cards. Find the probability that

both are kings


Answer:

Total number of all favorable cases is n(S) = 52


Let A be the event that first card drawn is a king. There are four kings in the pack. Hence, the probability of the first card is a king is



Let B be the event that second card is also king without replacement. Then there are 3 kings left in the pack as the cards are not replaced. Therefore, the probability of the second card is also king is



Then the probability of getting two kings without replacement is



(as there are 4 kings out of 52 cards in first draw, and 3 kings out of 51 cards in the second draw as the cards are not replaced)



The probability that both of them are kings is



Question 7.

Two cards are drawn without replacement from a pack of 52 cards. Find the probability that

the first is a king and the second is an ace


Answer:

Total number of all favorable cases is n(S) = 52


Let A be the event that first card drawn is a king. There are four kings in the pack. Hence, the probability of the first card is a king is



Let B be the event that second card is an ace without replacement. Then there are 4 aces in the pack as the cards are not replaced. Therefore, the probability of the second card is an ace is



Then the probability of getting first is a king and the second is an ace without replacement is



(as there are 4 kings out of 52 cards in first draw, and 4 aces out of 51 cards in the second draw as the cards are not replaced)



The probability that first is a king and the second is an ace without replacement is



Question 8.

Two cards are drawn without replacement from a pack of 52 cards. Find the probability that

the first is a heart and second is red.


Answer:

Total number of all favorable cases is n(S) = 52


Let A be the event that first card drawn is a heart. There are 13 hearts in the pack. Hence, the probability of the first card is a heart is



Let B be the event that second card is red without replacement. Then there are 26 red cards in the pack but as the cards are not replaced now there are 25 red cards as one heart which is red in color is already drawn out. Therefore, the probability of the second card is a red is



Then the probability of getting first is a heart and the second is a red card without replacement is





The probability that first is a heart and the second is red card without replacement is



Question 9.

A bag contains 20 tickets, numbered from 1 to 20. Two tickets are drawn without replacement. What is the probability that the first ticket has an even number and the second an odd number?


Answer:

There are 20 tickets numbered from 1 to 25, so the sample space is


S = {1, 2, 3, …., 20}, n(S) = 20


Number of even numbered tickets from 1 to 20 is


{2, 4, 6, 8, 10, 12, 14, 16, 18, 20} = 10


Number of odd numbered tickets from 1 to 20 is


{1, 3, 5, 7, 9, 11, 13, 15, 17, 19} = 10


Let A represents first ticket with even number and B represents second ticket with odd number


Then the probability of first ticket having even number and second ticket having odd number without replacement is



(as there are 10 even numbered tickets out of 20 tickets in first draw, and 10 odd numbered tickets out of 19 tickets in second draw as the tickets are not replaced)


Hence the required probability is



Question 10.

An urn contains 3 white, 4 red and 5 black balls. Two balls are drawn one by one without replacement. What is the probability that at least one ball is black.


Answer:

There are 3 white, 4 red and 5 black balls in the bag, so the number of all favorable outcomes in the sample space is


n(S) = 3+4+5=12


Let A be the event of not getting a black ball in the first draw, this means getting another color (red or white) ball out of 12 balls in the first draw. Hence the probability becomes


(as there are 3 white + 4 red = 7 balls)


Let B represents the event of not getting a black ball in the second draw, this means getting another color (red or white) ball out of 11 balls in the second draw as the balls are not replaced. Hence the probability becomes


(as there are 3 white + 4 red = 7 balls and one ball is already drawn in first draw so now there are total of 6 red and white balls)


Then the probability of at least one black ball without replacement







Hence the required probability is



Question 11.

A bag contains 5 white, 7 red and 3 black balls. If three balls are drawn one by one without replacement, find the probability that none is red.


Answer:

There are 5 white, 7 red and 3 black balls in the bag, so the number of all favorable outcomes in the sample space is


n(S) = 5+7+3=15


Let A be the event of getting a red ball in the first draw. Hence the probability becomes


(as there are 7 red balls out of 15 balls)


Let B represents the event of getting a red ball in the second draw. Hence the probability becomes


(as there are 7 red balls and one red ball is already drawn in first draw so now there are total of 6 red balls)


Let C represents the event of getting a red ball in the third draw. Hence the probability becomes


(as there are 7 red balls and two red balls are already drawn in first and second draw so now there are total of 5 red balls)


Then the probability of none being red balls without replacement







Hence the required probability is



Question 12.

A card is drawn from a well-shuffled deck of 52 cards and then a second card is drawn. Find the probability that the first card is a heart and the second card is diamond if the first card is not replaced.


Answer:

There are 52 cards in a deck. Let A be the event that first card drawn is a heart. There are 13 hearts in the pack. Hence, the probability of the first card is a heart is



Let B be the event that second card is a diamond without replacement. Then there are 13 diamond cards in the pack but as the cards are not replaced now there are 51 cards in total in the deck. Therefore, the probability of the second card is a red is



Then the probability of getting first is a heart and the second is a diamond card without replacement is





The probability that first is a heart and the second is diamond card without replacement is



Question 13.

An urn contains 10 black and 5 white balls. Two balls are drawn from the urn one after the other without replacement. What is the probability that both drawn balls are black?


Answer:

There are 10 black and 5 white balls in the bag, so the number of all favorable outcomes in the sample space is


n(S) = 10+5=15


Let A be the event of getting a black ball in the first draw. Hence the probability becomes


(as there are 10 black balls out of 15 balls)


Let B represents the event of getting a black ball in the second draw. Hence the probability becomes


(as there are 10 black balls and one black ball is already drawn in first draw so now there are total of 9 black balls)


Then the probability of all being black balls without replacement






Hence the required probability is



Question 14.

Three cards are drawn successively, without replacement from a pack of 52 well shuffled cards. What is the probability the first two cards are kings and third card drawn is an ace?


Answer:

There are 52 cards in a deck. Let A be the event that first card drawn is a king. There are 4 hearts in the pack. Hence, the probability of the first card is a king is



Let B be the event that second card is also a king without replacement. Then there are 3 king cards out of 51 cards in the pack as the cards are not replaced. Therefore, the probability of the second card is a red is



Let C be the event that third card is an ace card without replacement. Then there are 4 ace cards out of 50 cards in the pack as the cards are not replaced. Therefore, the probability of the third card is an ace card is



Then the probabilities of getting first two cards are kings and third card drawn is an ace without replacement is





The probability that first two cards are kings and third card drawn is an ace without replacement is



Question 15.

A box of oranges is inspected by examining three randomly selected oranges drawn without replacement. If all the three oranges are good, the box is approved for sale otherwise it is rejected. Find the probability that a box containing 15 oranges out of which 12 are good and 3 are bad ones will be approved for sale.


Answer:

Number of oranges the box contains = 15


Number of good oranges = 12


Number of bad oranges = 3


Probability that box is approved for sale


= Probability that first orange is good × probability that second orange is good, given first is good × probability that third orange is good, given first two are good


Let A represents a good orange


Then P(A)=P(getting first orange as good)


And


P(A│A)=P(getting second orange good, given first is good)


(as now there are 11 good oranges left out of 14 total oranges as one good orange is already drawn in first draw and are not replaced)


And also,


P(AA│A)=P(getting third orange good, given first two are good)


(as now there are 10 good oranges left out of 13 total oranges as two good orange is already drawn in first draw and are not replaced)


So the probability that box is approved for sale is




Therefore probability that box is approved for sale is



Question 16.

A bag contains 4 white, 7 black 5 red balls. Three balls are drawn one after the other without replacement. Find the probability that the balls drawn are white, black and red respectively.


Answer:

There are 4 white, 7 black and 5 red balls in the bag, so the number of all favorable outcomes in the sample space is


n(S) = 4+7+5=16


Let A be the event of getting a white ball in the first draw. Hence the probability becomes


(as there are 4 white balls out of 16 balls)


Let B represents the event of getting a black ball in the second draw. Hence the probability becomes


(as there are 7 black balls out of 15 balls as balls are not replaced back in the bag)


Let C represents the event of getting a red ball in the third draw. Hence the probability becomes


(as there are 5 red balls out of 14 balls as balls are not replaced back in the bag)


Then the probability that the balls drawn are white, black and red respectively without replacement






Hence the required probability is




Exercise 31.3
Question 1.

If P (A) = 7/13, P (B) =9/13 and P (A ∩ B) = 4/13, find p (A/B).


Answer:

We have, =


From the given data, P (A ∩ B) =


And P (B) =


Hence, =


= (answer)



Question 2.

If A and B are events such that P (A) =0.6, P (B) = 0.3 and P (A ∩ B) = 0.2, find P (A/B) and P(B/A).


Answer:

We have, =


And =


And also P (A ∩ B) = P (B ∩ A)


Hence = (answer)


And (answer)



Question 3.

If A and B are two events such that P (A ∩ B) = 0.32 and P (B) = 0.5, find P (A/B).


Answer:

We have, =


Therefore =


{multiply numerator and denominator by 2 to convert into the whole number}


= 0.64 (answer)



Question 4.

If P (A) = 0.4, P (B) = 0.8, P (B/A) = 0.6. Find P (A/B) and P (A ∪ B).


Answer:

We have, P (A ∪ B) = P (A)+ P (B)- P (A ∩ B)


From equation, =


P (B ∩ A) = × P (A) = 0.6× 0.4


= 0.24


Hence, = =


= 0.3 (answer)


And P (A ∪ B) = P (A)+ P (B)- P (A ∩ B)


= 0.4+0.8-0.24


= 0.96 (answer)



Question 5.

If A and B are two events such that

P (A) = 1/3, P (B)=1/4 and P(A ∪ B) = 5/12, find P (A/B) and P (B/A).


Answer:

We have, P (A) = , P (B)= and P(A ∪ B) =


We have, P (A ∪ B) = P (A)+ P (B)- P (A ∩ B)


Therefore, P (A ∩ B) = P (A)+ P (B) - P (A ∪ B)


= )


= {Make denominator same by taking LCM)


= (Answer)


Now, = = )


= (answer)


And = = = (answer)



Question 6.

If A and B are two events such that

P (A) = 6/11, P (B) = 5/11 and P ( A ∪ B) = 7/11, find P (A ∩ B), P (A/B), P (B/A)


Answer:

We have P (A) = 6/11, P (B) = 5/11 and P (A ∪ B) = 7/11


We have, P (A ∪ B) = P (A)+ P (B)- P (A ∩ B)


Therefore, P (A ∩ B) = P (A)+ P (B) - P (A ∪ B)


=


= { 11 is common in denominator }


= (answer)


Now, = =


= (answer)


And = =


(answer)



Question 7.

If A and B are two events such that

P (A)= 7/13, P (B)=9/13 and P(A ∩ B)=4/13, find P (A/B).


Answer:

We have, P (A)= 7/13, P (B)=9/13 and P(A ∩ B)=4/13

Now, =


= = (answer)1



Question 8.

If A and B are two events such that

P(A)=1/2,P (B)=1/3 and P(A ∩ B) = 1/4, find P (A/B), P(B/A), P (A/B) and P (A/B).


Answer:

We have, P(A)=1/2,P (B)=1/3 and P(A ∩ B) = 1/4


Now, =


=


= (answer)


And = =


(answer)



Question 9.

If A and B are two events such that 2P (A) = P (B) = 5/13 and P (A/B) = 2/5, find P (A ∪ B).


Answer:

We have, 2P (A) = P (B) =


2P (A) =


P (A) = =


Now, we need P (A ∩ B)


We have, =


P (A ∩ B) =


=


=


Now, P (A ∪ B) = P (A)+ P (B)- P (A ∩ B)


=


(taking lcm as 26 is common factor)


= (answer)



Question 10.

If P (A)=6/11,P (B)=5/11 and P (A ∪ B) = 7/11, find

i. P (A ∩ B)

ii. P (A /B)

iii. P (B/A)


Answer:

We have, P (A ∪ B) = P (A)+ P (B)- P (A ∩ B)


i. From above formula,


P (A ∩ B)= P (A)+ P (B) - P (A ∪ B)


=


= (answer)


ii. =


(answer)


iii. = = (answer)



Question 11.

A coin is tossed three times. Find P (A/B) in each of the following:

A=Heads on third toss, B=Heads on first two tosses


Answer:

When a coin is tossed three times, we have following outcomes


{HHH, HHT, HTH, THH, HTT, THT, TTH, TTT}


Total outcomes =8


From above outcomes,


A= Heads on third toss = {HHH, HTH, THH, TTH} =4


Therefore probability of occurrence of event A = P(A) =


B= heads on first two tosses = {HHH, HHT} =2


Therefore probability of occurrence of event B = P(B) =


Also we want P (A ∩ B) = probability of occurrence of both events A and B


=heads on first two tosses and heads on third toss


=heads on all tosses = {HHH} = 1(Occurrence Of both A and B events


Therefore, P (A ∩ B) =


Hence, =


==(answer)



Question 12.

A coin is tossed three times. Find P (A/B) in each of the following:

A=At least two heads, B=At most two heads


Answer:

When a coin is tossed three times, we have following outcomes


{HHH, HHT, HTH, THH, HTT, THT, TTH, TTT}


Total outcomes =8


A= at least two heads ={HHH, HHT, THH, HTH} = 4


Therefore probability of occurrence of event A = P(A) =


B= at most two heads ={HHT, HTH, HHT, THT, TTH, TTT, HTT} =7


Therefore probability of occurrence of event B = P(B) =


Now Also we want P (A ∩ B) = probability of occurrence of both events A and B


= occurrence of atleast and atmost two heads ={HHT, HTH, THH} =3


Hence, probability of occurrence of both events A and B = P (A ∩ B)=


Therefore, =


= (answer)



Question 13.

A coin is tossed three times. Find P (A/B) in each of the following:

A=At most two tails, B=At least one tail.


Answer:

When a coin is tossed three times, we have following outcomes


{HHH, HHT, HTH, THH, HTT, THT, TTH, TTT}


Total outcomes =8


A= atmost two tails ={HHH, HHT, HTH, THH, HTT, THT, TTH, } = 7


Therefore probability of occurrence of event A = P(A) =


B= at least one tail ={HHT, HTH, HHT, THT, TTH, TTT, HTT} =7


Therefore probability of occurrence of event B = P(B) =


Now Also we want P (A ∩ B) = probability of occurrence of both events A and B


= occurrence of atleast two tail and atleast one tail ={HHT, HTH, THH, HTT, THT, TTH,} =6


Hence, probability of occurrence of both events A and B = P (A ∩ B)=


Therefore, = = (answer)



Question 14.

Two coins are tossed once. Find P(A/B) in each of the following:

A=Tail appears on one coin, B= One coin shows head.


Answer:

When two coins are tossed, total outcomes are


{HH, HT, TH, TT} =4


A=Tail appears on one coin= {HT, TH} =2


Probability of A = P(A) =


B= One coin shows head ={HT, TH} =2


Probability of B =P(B) =


(A ∩ B) = one coin shows head and another shows tail = {HT, TH} =2


P (A ∩ B) =


Hence, =


= (answer)



Question 15.

Two coins are tossed once. Find P(A/B) in each of the following:

A=No tail appears, B=No head appears.


Answer:

When two coins are tossed, total outcomes are


{HH, HT, TH, TT} =4


A=No tail occurs = {HH} =1


Probability f A = P(A) =


B= No head occurs={TT} =1


Probability of B =P(B) =


(A ∩ B) = No tail and no head occurs = not possible =0


P (A ∩ B) =


Hence, =


= (answer)



Question 16.

A die is thrown three times. Find P(A/B) and P (B/A), if A=4 appears on the third toss, B=6 and 5 appear respectively on first two tosses.


Answer:

When a die is thrown 3 times, total possible outcomes are, 63 = 216


A= 4 appears on third toss =


{(1,1,4),(1,2,4),(1,3,4),(1,4,4),(1,5,4),(1,6,4)


(2,1,4),(2,2,4),(2,3,4),(2,4,4),(2,5,4),(2,6,4)


(3,1,4),(3,2,4),(3,3,4),(3,4,4),(3,5,4),(3,6,4)


(4,1,4), (4,2,4)(4,3,4),(4,4,4),(4,5,4),(4,6,4)


(5,1,4),(5,2,4),(5,3,4)(5,4,4),(5,5,4),(5,6,4)


(6,1,4),(6,2,4),(6,3,4),(6,4,4),(6,5,4),(6,6,4)}


=36


Hence P(A) =


B= 6,5 appears respectively on first two tosses


{(6,5,1),(6,5,2),(6,5,3),(6,5,4),(6,5,5),(6,5,6)}


= 6


Hence P(B) =


(A ∩ B) = 6,5 occurs on first two toss and 4 occur on third toss = (6,5,4) = 1


P (A ∩ B) =


Hence =


= = (answer)


= (answer)



Question 17.

Mother, father and son line up at random for a family picture. If A and B are two events given by A=son on one end, B= Father in the middle, find P (A/B) and P (B/A).


Answer:

Total possible outcomes =


{FMS, FSM, MFS, SMF, SFM, MSF} =6


A= Son on end = {FMS, MFS, SMF, SFM} =4


Therefore P(A) =


B= father in middle = {MFS, SFM} = 2


Therefore P(B) =


(A ∩ B) = son on one end and father in middle


= {MFS, SFM} =2


P(A ∩ B) =


Hence = (answer)


. = (answer)



Question 18.

A dice is thrown twice, and the sum of the numbers appearing is observed to be 6. What is the conditional probability that the number 4 has appeared at least once?


Answer:

When a dice is thrown 2 times, total outcomes are


{(1,1),(1,2),(1,3),(1,4),(1,5),(1,6)


(2,1),(2,2),(2,3),(2,4),(2,5),(2,6)


(3,1),(3,2),(3,3),(3,4),(3,5),(3,6)


(4,1),(4,2),(4,3),(4,4),(4,5),(4,6)


(5,1),(5,2),(5,3),(5,4),(5,5),(5,6)


(6,1),(6,2),(6,3),(6,4),(6,5),(6,6)}


= 36 possible outcomes


A= sum of numbers is 6


={(1,5),(2,4),(3,3),(4,2),(5,1)} = 5


P(A) =


B= 4 has appeared ateast once


= {(1,4),(2,4),(3,4),(4,1),(4,2),(4,3),(4,4),(4,5),(4,6),(5,4),(6,4)} =11


P(B)=


(A ∩ B) = sum of two number is 6 and also 4 has occurred ateast once


(A ∩ B) ={ (2,4),(4,2)} = 2


P(A ∩ B) =


Conditional probability = = (answer)



Question 19.

Two dice are thrown. Find the probability that the numbers appeared has the sum 8, if it is known that the second die always exhibits 4.


Answer:

We know total possible outcomes when two dice are thrown = 36


A= sum is 8


{(2,6),(3,5),(4,4),(5,3),(6,2)} = 5


P(A) =


B =second die exhibit 4


{(1,4),(2,4),(3,4),(4,4),(5,4),(6,4)}= 6


P(B) =


(A ∩ B) = sum of two number is 8 and also second die exhibit 4


=(4, 4) =1


P(A ∩ B) =


Therefore, = (answer)



Question 20.

A pair of dice is thrown. Find the probability of getting 7 as the sum, if it is known that the second die always exhibits an odd number.


Answer:

We know total possible outcomes when two dice are thrown = 36


A= sum is 7


{(1,6),(2,5),(3,4),(4,3),(5,2),(6,1l} = 6


P(A) =


B =second die exhibit odd number


{(1,1),(1,3),(1,5),(2,1),(2,3),(2,5),(3,1),(3,3),(3,5),(4,1),(4,3),(4,5),(5,1),(5,3),(5,5),(6,1),(6,3),(6,5)}= 18


P(B) =


(A ∩ B) = sum of two number is 7 and also second die exhibit odd number


={(2,5),(4,3),(6,1)} =3


P(A ∩ B) =


Therefore, =


= (answer)



Question 21.

A pair of dice is thrown. Find the probability of getting 7 as the sum if it is known that the second die always exhibits a prime number.


Answer:

We know, when a pair of dice is thrown, total possible outcomes are = 36


A=No of outcomes for getting 7 as sum are {(1,6),(2,5),(3,4),(4,3),(5,2),(6,1)} = 6


Therefore, P(A) = 6/36 =


B= No of outcomes for second die exhibiting prime no =


{(1,2),(1,3),(1,5),(2,2),(2,3),(2,5),(3,2),(3,3),(3,5),(4,2),(4,3),(4,5),(5,2),(5,3),(5,5),(6,2),(6,3),


(6,5)} = 18 {since 2,3,5 are prime nos )


Hence P(B) =


(A B) = outcomes of getting 7 as a sum with a second die showing prime no


= {(2,5), (5,2),(4,3)} = 3


Hence P(A B) = 3/36 =


Therefore, = = (answer)



Question 22.

A die is rolled. If the outcome is an odd number, what is the probability that it is prime?


Answer:

Total outcomes when a die is rolled = {1,2,3,4,5,6} = 6


A = outcome is odd number = {1,3,5} = 3


Hence P(A) =


B= Outcome is prime number = { 2,3,5) = 3


Hence P(B) =


(A B) = outcome is odd number and also prime number = {3,5} =2


Hence P(A B) =


We require, =


(answer)



Question 23.

A pair of dice is thrown. Find the probability of getting the sum 8 or more, if 4 appears on the first die.


Answer:

We know, when a pair of dice is thrown, total possible outcomes are = 36


A=No of outcomes for getting sum of 8 or more are


{(2,6),(3,5),(3,6),(4,4),(4,5),(4,6),(5,3),(5,4),(5,5),(5,6),(6,2),(6,3),(6,4),(6,5),(6,6)} = 15


Therefore, P(A) =


B= No of outcomes for first die showing 4 =


{(4,1),(4,2),(4,3),(4,4),(4,5),(4,6)} = 6


Hence P(B) =


(A B) = outcomes of getting sum as 8 or more with a first die showing 4


= {(4,4),(4,5),(4,6)} = 3


Hence P(A B) =


Therefore, = (answer)



Question 24.

Find the probability that the sum of the numbers showing on two dice is 8, given that least one die does not show five.


Answer:

We know, when a pair of dice is thrown, total possible outcomes are = 36


A=No of outcomes for getting sum of 8 are


{(2,6),(3,5),(4,4),(5,3),(6,2)} = 5


Therefore, P(A) =


B= No of outcomes for at least one die does not show 5


= {(1,1),(1,2),(1,3),(1,4),(1,6),(2,1),(2,2),(2,3),(2,4),(2,6),(3,1),(3,2),(3,3),(3,4),(3,6),(4,1),(4,2),


(4,3),(4,4),(4,6),(6,1),(6,2),(6,3),(6,4),(6,6) } = 25


Hence P(B) =


(A B) = outcomes of getting sum as 8 with at least one die not showing 5


= {(2,6),(4,4),(6,2)= 3


Hence P(A B) =


Therefore, =


(answer)



Question 25.

Two numbers are selected at random from integers 1 through 9 If the sum is even, find the probability that both the numbers are odd.


Answer:

Given: Two numbers are selected at random from integers 1 to 9.


Let A be the event when both the numbers are odd.


A = {(3, 1), (5, 1), (7, 1), (9, 1), (3, 5), (3, 7), (3, 9), (5, 3), (5, 7), (5, 9), (7, 3), (7, 5), (7, 9), (9, 3), (9, 5), (9, 7)}


Let B be the event when sum of both numbers is even.


B = {(1, 3(, (1, 5), (2, 4), (1, 7), (2, 6), (3, 5), (1, 9), (2, 8), (3, 7), (4, 6), (7, 5), (8, 4), (9, 3), (8, 6), (9, 5), (9, 7)}


A B = {(1, 3), (1, 5), (1, 7), (3, 5), (1, 9), (3, 7), (7, 5), (9, 3), (9, 5), (9, 7)}





Question 26.

A die is thrown twice, and the sum of the numbers appearing is observed to be 8 What is the conditional probability that the number 5 has appeared at least once?


Answer:

We know, when a pair of dice is thrown, total possible outcomes are = 36


A=No of outcomes for getting sum of 8 are


{(2,6),(3,5),(4,4),(5,3),(6,2)} = 5


Therefore, P(A) =


B= No of outcomes for at least 5 has appeared once =


= {(1,5),(2,5),(3,5),(4,5),(5,1),(5,2),(5,3),(5,4),(5,5),(5,6),(6,5)} =11


Hence P(B) =


(A B) = outcomes of getting sum as 8 with at least one die showing 5


= {(3,5),(5,3)= 2


Hence P(A B)


Therefore, =


(answer)



Question 27.

Two dice are thrownand it is known that the first die shows a 6 Find the probability that the sum of the numbers showing on two dice is 7


Answer:

Given: Two dice are thrown.


Let A be the event when the sum of the numbers = 7


Therefore, A = {(1, 6), (2, 5), (3, 4), (4, 3), (5, 2), (6, 1)}


Let B be the event when first dice shows 6


Therefore, B = {(6, 1), (6, 2), (6, 3), (6, 4), (6, 5), (6, 6)}


A B = {(6, 1)}





Question 28.

A pair of dice is thrown. Let E be the event that the sum is greater than or equal to 10 and F be the event”5 appears of the first-die”. Find P(E/F). If F is the event”5 appears on at least one die”, find P(E/F). (same as above question)


Answer:

We know, when a pair of dice is thrown, total possible outcomes are = 36

E=No of outcomes for getting sum greater than or equal to 10


{(4,6),(5,5),(5,6),(6,4),(6,5),(6,6)} = 6


Therefore, P(E) =


Case 1:


F= No of outcomes for 5 appears on first die =


= {(5,1),(5,2),(5,3),(5,4),(5,5),(5,6)} =6


Hence P(B) =


(E F ) = outcomes of getting a sum greater than or equal to 10 and first die showing 5


= {(5,6),(5,5)= 2


Hence P(E F) =


Therefore, = (answer)


Case 2:


F = 5 appears on atleast one die =


{(1,5),(2,5),(3,5),(4,5),( 5,1),(5,2),(5,3),(5,4),(5,5),(5,6),(6,5)} = 11


P(F) =


(E F ) = outcomes of getting a sum greater than or equal to 10 and at least die showing 5


= {(5,6),(5,5),(6,5)}= 3


Hence P(E F) =


Therefore, = (answer)



Question 29.

The probability that a student selected at random from a class will pass in Mathematics is 4/5, and probability that he/she passes in Mathematics and Computer Science is 1/2. What is the probability that he/she will pass in Computer Science if it is known that he/she has passed in Mathematics?


Answer:

Let M = Mathematics , C= computer science


By given data,


P(M) = and P(M C) =


We need to find, (Answer)



Question 30.

The probability that a certain person will buy a shirt is 0.2, the probability that he will buy a trouser is 0.3, and the probability that he will buy a shirt given that he will buy a trouser is 0.4. Find the probability that he will buy both a shirt and a trouser. Find also the probability that he will buy a trouser given that he buys a shirt.


Answer:

Let S= Shirt, T= Trouser


P(S) = 0.2 , P(T) = 0.3 and = 0.4


We need to find P(S T) and


We know, From given data, 0.4 = P (S T) / 0.3


P (S T) = = 0.12 (Answer).


Now we want,


= (answer)



Question 31.

In a school, there are 1000 students, out of which 430 are girls. It is known that out of 430, 10% of the girls study in XII. What is the probability that a student is chosen randomly studies in Class XII given that the chosen student is a girl?


Answer:

Given: Total Number of students = 1000


Number of Girls = 430


% of girls in Class XII = 10%


Let A be the event of Student chosen studies in Class XII


And Let B be the event that the student chosen is a girl


Now,





Question 32.

Ten cards numbered 1 through 10 are placed in a box, mixed up thoroughly and then one card is drawn randomly. If it is known that the number on the drawn card is more than 3, what is the probability that it is an even number?


Answer:

Given: Total Number of Cards = 10


Let A be the event of drawing a number more than 3.


Let B be the event of drawing an even number.




Question 33.

Assume that each born child is equally likely to be a boy or a girl. If a family has two children, what is the conditional probability that both are girls? Given that

i. the youngest is girl

ii. at least one is a girl.


Answer:

Let B= boy G= girl


And let us consider, in a sample space, the first child is elder and second child is younger.


Total possible outcome = {BB, BG, GB, GG} = 4


Let A= be the event that both the children are girls = 1


Therefore P(A) =


Case 1.


Let B = event that youngest is girl = {BG, GG} =2


{Since we have considered second is younger in a sample space}


Therefore P(B) =


And (A ∩ B) = both are girls and younger is also girl = (GG) = 1


Therefore , P (A ∩ B) =


We require


=


= (answer)


Case 2.


Let B = event that at least one is girl = {BG,GB GG} =3


{Since we have considered second is younger in a sample space}


Therefore P(B) =


And (A ∩ B) = both are girls and atlas one is girl = (GG) = 1


Therefore , P (A ∩ B) =


We require


=


= (answer)




Exercise 31.4
Question 1.

A coin is tossed thrice and all the eight outcomes are assumed equally likely. In which of the following cases are the following events A and B are independent?

i. A=the first throw results in head, B=the last throw results in tail

ii. A=the number of heads is odd, B=the number of tails is odd

iii. A=the number of heads is two, B=the last throw results in head


Answer:

It is given that the coin is tossed thrice, so the sample space will be,


S={HHH,HHT,HTH,HTT,THH,THT,TTH,TTT}


For independent event, P(A)*P(B)=P(A∩B)


i. A=the first throw results in head, B=the last throw results in tail


A={HHH,HHT,HTH,HTT}



B={HHT,HTT,THT,TTT}



A∩B={HHT,HTT}





Therefore A and B are independent events.


ii. A=the number of heads is odd, B=the number of tails is odd


A={HTT,THT,TTH}



B={HTH,THH,HHT}



A∩B={}=Ø



A and B are not independent


iii. A=the number of heads is two, B=the last throw results in head


A={HHT,HTH,THT}



B={HHH,HTH,THH,TTH}



A∩B={HTH}





Therefore A and B are not independent events.



Question 2.

Prove that is throwing a pair of dice, the occurrence of the number 4 on the first die is independent of the occurrence of 5 on the second die.


Answer:

We are throwing two dice so,


Sample space S={(1,1),(1,2),(1,3),(1,4),(1,5),(1,6),


(2,1),(2,2),(2,3),(2,4),(2,5),(2,6),


(3,1),(3,2),(3,3),(3,4),(3,5),(3,6),


(4,1),(4,2),(4,3),(4,4),(4,5),(4,6),


(5,1),(5,2),(5,3),(5,4),(5,5),(5,6),


(6,1),(6,2),(6,3),(6,4),(6,5),(6,6)}


Sample space contains 36 elements,


A= Number four appears on first die


A={(4,1),(4,2),(4,3),(4,4),(4,5),(4,6)}



B=Number 5 on second die


B={(1,5), (2,5), (3,5),(4,5),(5,5),(6,5)}



P(A∩B)={(4,5)}





Therefore A and B are independent events



Question 3.

A card is drawn form a pack of 52 cards so that each card is equally likely to be selected. In which of the following cases are the events A and N independent?

i. A=the card drawn is a king or queen, B=the card drawn is a queen or jack

ii. A=the card drawn is black, B=the card drawn is a king

iii. B=the card drawn is a spade, B=the card drawn in an ace


Answer:

(i) In a pack of 52 cards there are 4 king,4 queen,4 jack


A=the card drawn is a king or queen



B=the card drawn is a queen or jack



A∩B=Card is a queen





Therefore A and B are not independent events


(ii) In a pack of 52 cards there are 4 king,26 black


A=the card drawn is a black



B=the card drawn is a king



A∩B=Card is a black king





Therefore A and B are independent events


(iii) In a pack of 52 cards there are 13 spade,4 ace


A=the card drawn is a spade



B=the card drawn is an ace



A∩B=Card is an ace from spade





Therefore A and B are independent events



Question 4.

A coin is tossed three times. Let the events A,B and C be defined as follows:

A=first toss is head, B=second toss is head, and C= exactly two heads are tossed in a row.

Check the independence of (i) A and B

(ii) B and C

(iii) C and A


Answer:

Sample space S={(HHH),(HHT),(HTH),(HTT),(THH),(THT),(TTH),(TTT)}


A=first toss is head


A={HHH,HTH,HHT,HTT}


B=second toss is head


B={HHH,HHT,THT,THH}




A∩B={HHH,HHT}





Therefore A and B are independent events.


(ii) Sample space S={(HHH),(HHT),(HTH),(HTT),(THH),(THT),(TTH),(TTT)}


B=second toss is head


P(B)={HHH,HHT,THT,THH}


C= exactly two heads are tossed in a row.


P(C)={HHT,THH}




A∩B={HHT,THH}





Therefore B and C are not independent events.


(iii) Sample space S={(HHH),(HHT),(HTH),(HTT),(THH),(THT),(TTH),(TTT)}


C= exactly two heads are tossed in a row.


C={HHT,THH}


A=first toss is head


A={HHH,HTH,HHT,HTT}




A∩B={HHT}





Therefore A and C are independent events.



Question 5.

If A and B be two events such that P(A) = 1/4, P (B) = 1/3 and P (A ∪ B) = 1/2, show that A and B are independent events.


Answer:

P (A ∪ B)=P(A)+P(B)–P(A∩B)







Therefore A and B are independent events.



Question 6.

Given two independent events A and B such that P (A)=0.3 and P(B) = 0.6 Find

i. P (A B)

ii.

iv.

iv. P (A ∪ B)

v. P (A / B)

vi. P (B / A)


Answer:

For independent event,



P(A)*P(B)=0.3*0.6=0.18


ii.




iii.




iv.


=[(1–P(A)][1–P(B)] =[1–0.3][1–0.6]


=0.7*0.4=0.28


iv. P (A ∪ B)


P (A ∪ B)=P(A)+P(B)–P(A∩B)


=0.3+0.6–0.18=0.72


v. P (A / B)



vi. P (B / A)




Question 7.

If P (not B) = 0.65, P (A ∪ B) = 0.85, and A and B are independent events, then find p (A).


Answer:

We know that,


P (A ∪ B)=P(A)+P(B)–P(A∩B)


P(not B)=1–P(B)


=1–0.65=0.35


Given that A and B are independent events,



Therefore,


P (A ∪ B)=P(A)+P(B)–P(A∩B)


=P(A)+P(B)–[P(A)*P(B)]


P (A ∪ B)=P(A)[1–P(B)]+P(B)


0.85=P(A)*0.65+0.35


P(A)*0.65=0.50




Question 8.

If A and B are two independent events such that =2/15 and =1/6, then find P(B).


Answer:

Let , denote the complements of A, B respectively.


Given that, P(A∩) )


P(A)P()


P(A)[1–P(B)]


P(A)=1/6[1–P(B)]


P(A'∩B)


P(A')P(B)


[1–P(A)]P(B) )


[1–1/6{1–P(B)}]P(B) )


[{6–6P(B)–1}/{6–6P(B)}]P(B) )


15[5–6P(B)]P(B)=2[6–6P(B)]


15[5–6P(B)]P(B)=12[1–P(B)]


5[5–6P(B)]P(B)=4[1–P(B)]


25P(B)–30[P(B)]2=4–4P(B)


–30[P(B)]2+25P(B)+4P(B)–4=0


30[P(B)]2–29P(B)+4=0


30a2–29a+4=0 where P(B)=a


30a2–24a–5a+4=0


6a(5a–4)–1(5a–4)=0


(6a–1)(5a–4)=0


6a–1=0


6a=1


a


P(B)


5a–4=0


5a=4


a=


P(B)


Therefore, P(B),



Question 9.

A and B are two independent events. The probability that A and B occur is 1/6 and the probability that neither of them occurs is 1/3. Find the probability of occurrence of two events.


Answer:

P(A∩B)=









6P(A)2–5P(A)+1=0


6P(A)2–3P(A)–2P(A)+1=0


[2P(A)–1][3P(A)–1]=0


2P(A)–1=0


2P(A)=1



3P(A)–1=0


3P(A)=1





Question 10.

If A and B are two independent events such that P (A ∪ B) = 0.60 and P(A)= 0.2, find P(B).


Answer:

Given that A and B are independent events,



We know that,


P (A ∪ B)=P(A)+P(B)–P(A∩B)


Therefore,


P (A ∪ B)=P(A)+P(B)–P(A∩B)


=P(A)+P(B)–[P(A)*P(B)]


0.6=0.2+P(B)–0.2P(B)


0.6–0.2=P(B)[1–0.2]


0.4=P(B)*0.8




Question 11.

A die Is tossed twice. Find the probability of getting a number greater than 3 on each toss.


Answer:

We are throwing two dice so,


Sample space S={(1,1),(1,2),(1,3),(1,4),(1,5),(1,6),


(2,1),(2,2),(2,3),(2,4),(2,5),(2,6),


(3,1),(3,2),(3,3),(3,4),(3,5),(3,6),


(4,1),(4,2),(4,3),(4,4),(4,5),(4,6),


(5,1),(5,2),(5,3),(5,4),(5,5),(5,6),


(6,1),(6,2),(6,3),(6,4),(6,5),(6,6)}


Sample space contains 36 elements,


Let ,


A= getting a number greater than 3 on first toss


A={(4,1),(4,2),(4,3),(4,4),(4,5),(4,6),


(5,1),(5,2),(5,3),(5,4),(5,5),(5,6),


(6,1),(6,2),(6,3),(6,4),(6,5),(6,6)}



B= getting a number greater than 3 on second toss


B={(1,4),(1,5),(1,6), (2,4),(2,5),(2,6), (3,4),(3,5),(3,6), (4,4),(4,5),(4,6), (5,4),(5,5),(5,6), (6,4),(6,5),(6,6)}



P(A∩B)=P(getting a number on each toss)


Given that A and B are independent events,





Question 12.

Given the probability that A can solve a problem is 2/3 and the probability that B can solve the same problem is 3/5. /find the probability that none of the two will be able to solve the problem.


Answer:

Given that P(A can solve problem)=




P(B can solve the problem)=




P(none can solve the problem)=


Therefore, required probability=



Question 13.

An unbiased die is tossed twice. Find the probability of getting 4,5, or 6 on the first toss and 1,2,3 or 4 on the second toss.


Answer:

Given an unbiased die is tossed twice so the sample space contain 36 elements.


Let A be the probability of getting 4,5,6 on first toss


A={(4,1),(4,2),(4,3),(4,4),(4,5),(4,6),


(5,1),(5,2),(5,3),(5,4),(5,5),(5,6),


(6,1),(6,2),(6,3),(6,4),(6,5),(6,6)}



Let B be the probability of getting 1,2,3,4 on the second toss


B={1,1),(1,2),(1,3),(1,4),(2,1),(2,2),(2,3),(2,4),


(3,1),(3,2),(3,3),(3,4),(4,1),(4,2),(4,3),(4,4),(5,1),(5,2),(5,3),(5,4)


(6,1),(6,2),(6,3),(6,4)}



P(getting 4,5,6 on the first toss and 1,2,3 or 4 on the second toss)





Question 14.

A bag contains 3 red and 2 black balls. One ball Is drawn from it at random. Its colour is noted and then it is put back in the bag. A second draw is made and the same procedure is repeated. Find the probability of drawing (i) tow red balls, (ii) two black balls, (iii) first red and second black ball.


Answer:

Given, bag contains 3 red and 2 black balls.


Probability of getting red ball=


Probability of getting black ball=


(i) two red balls


P(getting two getting red balls)=P(A)*P(A)



(ii) two black balls


P(getting two black balls)=P(B)*P(B)



(iii) first red and second black ball.


P( getting first red and second black ball)=P(A)*P(B)




Question 15.

Three cards are drawn with replacement from a well shuffled pack of cards. Find the probability that the cards drawn are king, queen and jack.


Answer:

In a pack of 52 cards, there are 4 king,4 queen and 4 jack


Let A be drawn card is a king



Let B be drawn card is a queen



Let C be drawn card is a jack



P(cards drawn are king, queen, jack )=


=P(A)P(B)P(C)+P(A)P(C)P(B)+P(B)P(A)P(C)+P(B)P(C)P(A)+P(C)P(A)P(B)+P(C)P(B)P(A)


=6*P(A)P(B)P(C)



Question 16.

An article manufactured by a company consists of two parts X and Y. In the process of manufacture of the part X, 9 out of 100 parts may be defective. Similarly, 5 out of 100 are likely to be defective in the manufacture of part Y. Calculate the probability that the assembled product will not be defective.


Answer:

Given,


An article manufactured by a company consists of two parts X and Y.


Part X has 9 out of 100 defective


That is, Part X has 91 out of 100 non defective


Part Y has 5 out of 100 defective


That is, Part Y has 95 out of 100 non defective


Consider,


X=a non defective part of X


Y=a non defective part of Y



P(assembled product will not be defective)


P(neither X defective nor Y defective)


=P(X∩Y)


=P(X)P(Y)




Question 17.

The probability that A hits a target is 1/3 and the probability that B hits it, is 2/5. What is the probability that the target will be hit, if each one of A and B shoots at the target?


Answer:

Given,


Probability that A hits a target is 1/3


P(A)=


The probability that B hits it, is 2/5


P(b)=


P(target will be hit)=1–P(target will not be hit)


=1–P(neither A nor B hit the target)








Question 18.

An anti–aircraft gun can take a maximum of 4 shots at an enemy plane moving away from it. The probabilities of hitting the plane at the first, second, third and fourth shot are 0.4, 0.3, 0.2 and 0.1 respectively. What is the probability that the gun hits the plane?


Answer:

Given that an anti–aircraft gun can take a maximum of 4 shots at an enemy plane moving away from it.


Let


A=hitting the plane at first shot


P(A)=0.4


B=hitting the plane at second shot


P(B)=0.3


C=hitting the plane at third shot


P(C)=0.2


D=hitting the plane at fourth shot


P(D)=0.1


P(gun hits the plane)=1–P(gun does not hit the plane)


=1–P(none of the four shots hit the plane)





=1–[(1–0.4)(1–0.3)(1–0.2)(1–0.1)]


=1–(0.6)(0.7)(0.8)(0.9)


=1–0.3024=0.6976



Question 19.

The odds against a certain event are 5 to 2 and the odds in favour of another event, independent to the former are 6 to 5. Find the probability that (i) at least one of the events will occur, and (ii) none of the events will occur.


Answer:

(i) The odds against certain event are 5 to 2



odds in favour of another event are 6 to5




Find the probability that (i) at least one of the events will occur


=1–P(none of the events will occur)




(ii) none of the events will occur.


P(none of the events will occur)





Question 20.

A die is thrown thrice. Find the probability of getting an odd number at least once.


Answer:

The sample space has 216 outcomes.


A be getting an odd number in a throw of die



P(getting an odd number at least once)=1–P(getting no odd number)






Question 21.

Two balls are drawn at random with replacement from a box containing 10 black and 8 red balls. Find the probability that (i) both balls are red, (ii) first balls is black and second is red, (iii) one of them is black and other is red.


Answer:

Given an urn contains 10 black balls and 8 red balls


Probability of getting red ball=


Probability of getting black ball=


(i) both balls are red


P(getting two getting red balls)=P(R)*P(R)



(ii) first balls is black and second is red


P(first balls is black and second is red)=P(B)*P(R)


(iv) one of them is black and other is red.


P(one of them is black and other is red)=P(B)P(R)+P(R)P(B)




Question 22.

An urn contains 4 red and 7 black balls. Two balls are drawn at random with replacement. Find the probability of getting (i) 2 red balls, (ii) 2 black balls, (iii) one red and one black ball.


Answer:

Given an urn contains 4 red and 7 black balls


Probability of getting red ball=


Probability of getting black ball=


(i) two red balls


P(getting two getting red balls)=P(R)*P(R)



(ii) two black balls


P(getting two blue balls)=P(B)*P(B)



(iii) first red and second black ball.


P( getting one red and one black ball)=P(R)*P(B)+P(B)*P(R)




Question 23.

The probabilities of two students A and B coming to the school in time are 3/7 and 5/7 respectively. Assuming that the events, ‘A coming in time’ and ‘B coming in time’ are independent, find the probability of only one of them coming to the school in time. Write at least one advantage of coming to school in time.


Answer:

Given that A coming in time and B coming in time are independent.


Let A denote A coming in time


B denote B coming in time


denotes complementary event of A



denotes complementary event of B



P(only one coming in time)





Question 24.

Two dice are thrown together and the total score is noted. The event E, F and G are “a total 4”, “a total of 9 or more”, and “a total divisible by 5”, respectively. Calculate P(E), P(F) and P(G) and decide which pairs of events, if any, are independent.


Answer:

We are throwing two dice so,


Sample space S={(1,1),(1,2),(1,3),(1,4),(1,5),(1,6),


(2,1),(2,2),(2,3),(2,4),(2,5),(2,6),


(3,1),(3,2),(3,3),(3,4),(3,5),(3,6),


(4,1),(4,2),(4,3),(4,4),(4,5),(4,6),


(5,1),(5,2),(5,3),(5,4),(5,5),(5,6),


(6,1),(6,2),(6,3),(6,4),(6,5),(6,6)}


Sample space contains 36 elements,


n(S)=36


let E be an event of getting sum 4


E={(1,3),(3,1),(2,2)}


n(E)=3



F be the event of getting 9 or more


F ={(3,6),(6,3),(4,5),(5,4),(4,6),(6,4),(5,5),(5,6),(6,5),(6,6)}


n(F)=3



G be the event getting a total divisible by 5


G= {(1,4),(4,1),(2,3),(3,2),(4,6),(6,4),(5,5)}


n(G)=7



So E and F are not independent


So E and G are not independent




So F and G are not independent


Therefore no pair is independent



Question 25.

Let A and B be two independent events such that P(A) = p1 and P(B) = p2. Describe in words the events whose probabilities are:

i. p1p2

ii. (1–p1) p2

iii. 1–(1–p1)(1–p2)

iv. p1+p2 = 2p1p2


Answer:

If the events are said to be independent, if the occurrence or non occurrence of one does not affect the probability of the occurrence or non occurrence of other.


i. p1p2


p1p2=P(A)P(B)


Both events A and B will occur


ii. (1–p1) p2


(1–p1) p2=[1–P(A)]P(B)



Event A does not occur but event B occur.


iii. 1–(1–p1)(1–p2)


1–(1–p1)(1–p2)=[1–(1–P(A))(1–P(B))]



At least one of the event will occur


iv. p1+p2 = 2p1p2


p1+p2 = 2p1p2


P(A)+P(B)=2P(A)P(B)


P(A)+P(B)–2P(A)P(B)=0


P(A)–P(A)P(B)+P(B)–P(A)P(B)=0


P(A)[1–P(B)]+P(B)[1–P(A)]=0




Exactly one of A and B occurs




Exercise 31.5
Question 1.

A bag contains 6 black and 3 white balls. Another bag contains 5 black and 4 white balls. If one ball is drawn from each bag, find the probability that these two balls are of the same colour.


Answer:

Given:


⇒ Bag A contains 6 black balls and 3 white balls


⇒ Bag B contains 5 black balls and 4 white balls


It is told that one ball is drawn is drawn from is each bag.


We need to find the probability that the balls are of the same colour.


Let us find the Probability of drawing each colour ball from the bag.


⇒ P(B1) = P(drawing black ball from bag A)





⇒ P(W1) = P(drawing white ball from bag A)





⇒ P(B2) = P(drawing black ball from bag B)





⇒ P(W2) = P(drawing white ball from bag B)





We need to find the probability of drawing the same colour balls from two bags


⇒ P(S) = P(drawing two balls of same colours) = P(drawing black balls from each bag) + (P(drawing white balls from each bag)


Since drawing a ball is independent for each bag, the probabilities multiply each other.





.


∴ The required probability is .



Question 2.

A bag contains 3 red and 5 black balls and a second bag contains 6 red and 4 black balls. A ball is drawn from each bag. Find the probability that one is red and the other is black.


Answer:

Given:


⇒ Bag A contains 3 red balls and 5 black balls


⇒ Bag B contains 6 red balls and 4 black balls


It is told that one ball is drawn is drawn from is each bag.


We need to find the probability that one ball is red and other is black.


Let us find the Probability of drawing each colour ball from the bag.


⇒ P(B1) = P(drawing black ball from bag A)





⇒ P(R1) = P(drawing Red ball from bag A)





⇒ P(B2) = P(drawing black ball from bag B)





⇒ P(R2) = P(drawing Red ball from bag B)





We need to find the probability of drawing the different colour balls from two bags


⇒ P(S) = P(drawing one red ball and one Black ball)


⇒ P(S) = P(drawing black balls from bag A and red ball from bag B) + P(drawing black balls from bag B and red ball from bag A)


Since drawing a ball is independent for each bag, the probabilities multiply each other.





.


∴ The required probability is .



Question 3.

Two balls are drawn at random with replacement from a box containing 10 black and 8 red balls. Find the probability that (i) both the balls are red. (ii) first ball is black and second is red. (iii) one of them is black and other is red.


Answer:

Given:


⇒ Bag contains 10 black balls and 8 red balls


It is told that two balls are drawn from bag with replacement.


Let us find the Probability of drawing each colour ball from the bag.


⇒ P(B1) = P(drawing black ball from bag)





⇒ P(R1) = P(drawing red ball from bag)





We need to find:


(i) P(Drr) = P(both balls drawn are red)


(ii) P(Dbr) = P(first drawn is black and next is red)


(iii) P(Srd) = P(one ball is red and other is black)


⇒ P(Drr) = P(both balls drawn are red)


Since drawing a ball is independent for each bag, the probabilities multiply each other.





⇒ P(Dbr) = P(first drawn is black and next is red)


Since drawing a ball is independent for each bag, the probabilities multiply each other.





⇒ P(Srb) = P(one ball is red and other is black)


⇒ P(Srb) = P(first drawn is red and next is black) + P(first drawn black and next is red)


⇒ P(Srb) = P(Drb) + P(Dbr)


Since drawing a ball is independent for each bag, the probabilities multiply each other.






∴ The required probabilities are .



Question 4.

Two cards are drawn successively without replacement from a well-shuffled deck of 52 cards. Find the probability of exactly one ace.


Answer:

Given that two cards are drawn from a well-shuffled deck of 52 cards.


We know that there will 4 aces present in a deck.


It is told that two cards successively without replacement.


Let us find the probability of drawing the cards.


⇒ P(A1) = P(Drawing ace from 52 cards deck)




⇒ P(O1) = P(Drawing cards other than ace from 52 cards deck)




⇒ P(A2) = P(Drawing ace from remaining 51 cards deck)




⇒ P(O2) = P(Drawing a card other than ace from remaining 51 cards deck)




We need to find the probability of drawing exactly one ace


⇒ P(DA) = P(Drawing exactly 1 ace in the drawn two cards)


⇒ P(DA) = P(Drawing Ace first and others next) + (P(Drawing Other cards first and ace next)


Since drawing cards are independent their probabilities multiply each other,


⇒ P(DA) = (P(A1)P(O2)) + (P(O1)P(A2))





∴ The required probability is .



Question 5.

A speaks truth in 75% and B in 80% of the cases. In what percentage of cases are they likely to contradict each other in narrating the same incident?


Answer:

Given:


⇒ A speaks truth in 75% of cases


⇒ B speaks truth in 80% of cases


Now,


⇒ P(TA) = P(A speaking truth) = 0.75


⇒ P(NA) = P(A not speaking truth) = 1-0.75


⇒ P(NA) = 0.25


⇒ P(TB) = P(B speaking truth) = 0.80


⇒ P(NB) = P(B not speaking truth) = 1-0.80


⇒ P(NB) = 0.20


We need to find the probability for case in which A and B contradict each other for narrating an incident.


This happens only when A not telling truth while B is telling truth and vice-versa.


⇒ P(CAB) = P(A and B contradict each other)


⇒ P(CAB) = P(A tells truth and B doesn’t) + P(B tells truth and A doesn’t)


Since the speaking of A and B are independent events their probabilities will multiply each other.


⇒ P(CAB ) = (P(TA)P(NB)) + (P(NA)P(TB))


⇒ P(CAB) = (0.75×0.20) + (0.25×0.80)


⇒ P(CAB) = 0.15 + 0.20


⇒ P(CAB) = 0.35


∴ The required probability is 0.35.



Question 6.

Kamal and Monica appeared for an interview for two vacancies. The probability of Kamal’s selection is 1/3 and that of Monika’s selection is 1/5. Find the probability that

i. Both of them will be selected

ii. None of them will be selected

iii. At least one of them will be selected

iv. Only one of them will be selected.


Answer:

Given that,


⇒ P(KS) = P(Kamal’s selection)



⇒ P(KN) = P(Not selecting Kamal)




⇒ P(VS) = P(Monika’s selection)



⇒ P(VN) = P(Not selecting Monika)




We need to find:


i. Both of them will be selected


ii. None of them will be selected


iii. At least one of them will be selected


iv. Only one of them will be selected.


⇒ P(Sboth) = P(Both of them are selected)


Since selection of each person is an independent event their probabilities multiply each other





⇒ P(Snone) = P(None of them are selected)


Since selection of each person is an independent event their probabilities multiply each other





⇒ P(Satone) = P(Selecting at least one of them)


⇒ P(Satone) = P(selecting only Kamal) + P(selecting only Monika) + P(Selecting both)


Since selection of each person is an independent event their probabilities multiply each other






⇒ P(Sone) = P(Only one of them is selected)


⇒ P(Sone) = P(only Kamal is selected) + P(only Monika is selected)


Since selection of each person is an independent event their probabilities multiply each other





∴ The required probabilities are .



Question 7.

A bag contains 3 white, 4 red and 5 black balls. Two balls are drawn one after the other, without replacement. What is the probability that one is white and the other is black?


Answer:

Given:


⇒ Bag contains 3 white, 4 red and 5 black balls.


It is told that two balls are drawn from bag without replacement.


Let us find the Probability of drawing each colour ball from the bag.


⇒ P(B1) = P(drawing black ball from bag on first draw)





⇒ P(W1) = P(drawing white ball from bag on first draw)





⇒ P(B2) = P(drawing black ball from bag on second draw)




⇒ P(W2) = P(drawing white ball from bag on second draw)




We need to find:


⇒ P(SWB) = P(one ball is White and other is black)


⇒ P(SWB) = P(first drawn is White and next is black) + P(first drawn black and next is white)


⇒ P(SWB) = P(DWB) + P(DBW)


Since drawing a ball is independent for each bag, the probabilities multiply each other.







∴ The required probability is .



Question 8.

A bag contains 8 red and 6 green balls. Three balls are drawn one after another without replacement. Find the probability that at least two balls drawn are green.


Answer:

Given:


Bag contains 8 red and 6 green balls


It is told that three balls are drawn without replacement.


We need to find the probability that at least two balls drawn are green.


The possible cases of drawing balls are as shown below:


i. Green,Green,Red


ii. Green,Red,Green


iii. Red,Green,Green


iv. Green,Green,Green


⇒ P(Dat2greens) = P(GGR) + P(GRG) + P(RGG) + P(GGG)


Since drawing balls are independent events so, the probabilities multiply each other.







∴ The required probability is .



Question 9.

Arun and Tarun appeared for an interview for two vacancies. The probability of Arun’s selections is 1/4 and that of Tarun’s rejection is 2/3. Find the probability that at least one of them will be selected.


Answer:

Given that,


⇒ P(AS) = P(Arun’s selection)



⇒ P(AN) = P(Arun’s rejection)




⇒ P(TN) = P(Tarun’s rejection)



⇒ P(TS) = P(Tarun’s selection)




We need to find the probability that at least one of them will be selected


⇒ P(Satone) = P(Selecting at least one of them)


⇒ P(Satone) = P(selecting only Arun) + P(selecting only Tarun) + P(Selecting both)


Since selection of each person is an independent event their probabilities multiply each other






∴ The required probability is .



Question 10.

A and B toss a coin alternately till one of them gets a head and wins the game. If A starts the game, find the probability that B will win the game.


Answer:

Given that A and B toss a coin until one of them gets a head to win the game.


Let us find the probability of getting the head.


⇒ P(AH) = P(A getting a head on tossing a coin)



⇒ P(AN) = P(A not getting head on tossing a coin)



⇒ P(BH) = P(B getting a head on tossing a coin)



⇒ P(BN) = P(B not getting head on tossing a coin)



It is told that A starts the game.


We need to find the probability that B wins the games.


B wins the game only when A losses in1st,3rd,5th,…… tosses.


This can be shown as follows:


⇒ P(WB) = P(B wins the game)


⇒ P(WB) = P(ANBH) + P(ANBNANBH) + P(ANBNANBNANBH) + …………………


Since tossing a coin by each person is an independent event, the probabilities multiply each other.


⇒ P(WB) = (P(AN)P(BH)) + (P(AN)P(BN)P(AN)P(BH)) + ………………




The terms in the bracket resembles the infinite geometric series sequence:


We know that the sum of a Infinite geometric series with first term ‘a’ and common ratio ‘r’ is






∴ The required probability is .



Question 11.

Two cards are drawn from a well shuffled pack of 52 cards, one after another without replacement. Find the probability that one of these is red card and the other a black card?


Answer:

Given that two cards are drawn from a well-shuffled deck of 52 cards.


We know that there will 26 Red and 26 Black cards present in a deck.


It is told that two cards successively without replacement.


Let us find the probability of drawing the cards.


⇒ P(R1) = P(Drawing Red card from 52 cards deck)




⇒ P(B1) = P(Drawing Black card from 52 cards deck)




⇒ P(R2) = P(Drawing Red card from remaining 51 cards deck)




⇒ P(B2) = P(Drawing Black card from remaining 51 cards deck)




We need to find the probability of drawing exactly one Red and one black card


⇒ P(DRB) = P(Drawing exactly one red and one black card)


⇒ P(DRB) = P(Drawing Red first and Black next) + (P(Drawing Black first and red next)


Since drawing cards are independent their probabilities multiply each other,


⇒ P(DRB) = (P(R1)P(B2)) + (P(B1)P(R2))





∴ The required probability is .



Question 12.

Tickets are numbered from 1 to 10. Two tickets are drawn one after the other at random. Find the probability that the number on one of the tickets is a multiple of 5 and on the other a multiple of 4.


Answer:

Given that tickets are numbered from 1 to 10.


It is told that two tickets are drawn one after other in random.


We need to find the probability that number on ticket is a multiple of 5 and other number is multiple of 4.


Let us find the individual probabilities first


⇒ P(T51) = P(Number on ticket is multiple of 5 in 1st draw)





⇒ P(T41) = P(Number on ticket is multiple of 4 in 1st draw)





⇒ P(T52) = P(Number on ticket is multiple of 5 in 2nd draw)




⇒ P(T42) = P(Number on ticket is multiple of 4 in 2nd draw)




⇒ P(D54) = P(Drawing one ticket which is multiple of 5 and other is multiple of 4)


⇒ P(D54) = P(Drawing 5 multiple ticket first and 4 multiple ticket next) + (P(Drawing 4 multiple ticket first and 5 multiple ticket next)


Since drawing tickets are independent their probabilities multiply each other,


⇒ P(D54) = (P(T51)P(T42)) + (P(T41)P(T52))





∴ The required probability is .



Question 13.

In a family, the husband tells a lie in 30% cases and the wife in 35% cases. Find the probability that both contradict each other on the same fact.


Answer:

Given:


⇒ Husband tells lies in 30% of cases


⇒ Wife tells lies in 35% of cases


Now,


⇒ P(NH) = P(Husband telling lies) = 0.30


⇒ P(TH) = P(Husband telling truth) = 1-0.30


⇒ P(TH) = 0.70


⇒ P(NW) = P(Wife telling lies) = 0.35


⇒ P(TW) = P(Wife telling truth) = 1-0.35


⇒ P(TW) = 0.65


We need to find the probability for case in which husband and wife contradict each other for narrating a fact.


This happens only when husband not telling truth while Wife is telling truth and vice-versa.


⇒ P(CHW) = P(Husband and Wife contradict each other)


⇒ P(CHW) = P(Husband tells truth and Wife doesn’t) + P(Wife tells truth and Husband doesn’t)


Since the speaking of husband and Wife are independent events their probabilities will multiply each other.


⇒ P(CHW ) = (P(TH)P(NW)) + (P(NH)P(TW))


⇒ P(CHW) = (0.70×0.35) + (0.30×0.65)


⇒ P(CHW) = 0.245 + 0.195


⇒ P(CHW) = 0.44


∴ The required probability is 0.44.



Question 14.

A husband and wife appear in an interview for two vacancies for the same post. The probability of husband’s selection is 1/7 and that of wife’s selection is 1/5. What is the probability that

i. Both of them will be selected

ii. Only one them will be selected

iii. None of them will be selected


Answer:

Given that,


⇒ P(HS) = P(Husband’s selection)



⇒ P(HN) = P(Not selecting Husband)




⇒ P(WS) = P(wife’s selection)



⇒ P(WN) = P(Not selecting Wife)




We need to find:


i. Both of them will be selected


ii. Only one of them will be selected


iii. None of them will be selected


⇒ P(Sboth) = P(Both of them are selected)


Since selection of each person is an independent event their probabilities multiply each other





⇒ P(Sone) = P(Only one of them is selected)


⇒ P(Sone) = P(only Husband is selected) + P(only Wife is selected)


Since selection of each person is an independent event their probabilities multiply each other





⇒ P(Snone) = P(None of them are selected)


Since selection of each person is an independent event their probabilities multiply each other





∴ The required probabilities are .



Question 15.

A bag contains 7 white, 5 black and 4 red balls. Four balls are drawn without replacement. Find the probability that at least three balls are black.


Answer:

Given:


Bag contains 7 white, 5 black and 4 red balls


It is told that four balls are drawn without replacement.


We need to find the probability that at least three balls drawn are Black.


The possible cases of drawing balls are as shown below:


i. Black, Black, Black, Others


ii. Black, Black, Others, Black


iii. Black, Others, Black, Black


iv. Others, Black, Black, Black


v. Black, Black, Black, Black


⇒ P(Dat3blacks) = P(BBBO) + P(BBOB) + P(BOBB) + P(OBBB) + P(BBBB)


Since drawing balls are independent events so, the probabilities multiply each other.







∴ The required probability is .



Question 16.

A, B, and C are independent witness of an event which is known to have occurred. A speaks the truth three times out of four, B four times out of five and C five times out of six. What is the probability that the occurrence will be reported truthfully by majority of three witnesses?


Answer:

Given:


A speaks truth three times out of four


B speaks truth four times out of five


C speaks truth five times out of six


⇒ P(TA) = P(A speaks truth)



⇒ P(TB) = P(B speaks truth)



⇒ P(TC) = P(C speaks truth)



⇒ P(NA) = P(A speaks lies)




⇒ P(NB) = P(B speaks lies)




⇒ P(NC) = P(C speaks lies)



.


It is told that the occurrence is will be reported by majority of witness. This is only possible when at least two of the witnesses speaks truth.


⇒ P(M) = P(TATBNC) + P(TANBTC) + P(NATBTc) + P(TATBTC)


Since speaking by a person is independent the probabilities will multiply each other.


⇒ P(M) = (P(TA)P(TB)P(NC)) + (P(TA)P(NB)P(TC)) + (P(NA)P(TB)P(Tc)) + (P(TA)P(TB)P(TC))




.


∴ The required probability is .



Question 17.

A bag contains 4 white balls and 2 black balls. Another contains 3 white balls and 5 black balls. If one ball is drawn from each bag, find the probability that

i. Both are white

ii. Both are black

iii. One is white and one is black


Answer:

Given:


⇒ Bag A contains 4 white balls and 2 black balls


⇒ Bag B contains 3 white balls and 5 black balls


It is told that one ball is drawn is drawn from is each bag.


Let us find the Probability of drawing each colour ball from the bag.


⇒ P(B1) = P(drawing black ball from bag A)





⇒ P(W1) = P(drawing white ball from bag A)





⇒ P(B2) = P(drawing black ball from bag B)





⇒ P(W2) = P(drawing white ball from bag B)





We need to find:


i. P(DWW) = P(Both are white)


ii. P(DBB) = P(Both are black)


iii. P(DWB) = P(One drawn is white and other is black)


⇒ P(DWW) = P(Both are White)


Since drawing a ball is independent for each bag, the probabilities multiply each other.





P(DBB) = P(Both balls are black)


Since drawing a ball is independent for each bag, the probabilities multiply each other.





P(DWB) = P(One ball drawn is white and other is black)


Since drawing a ball is independent for each bag, the probabilities multiply each other.





.


∴ The required probabilities are .



Question 18.

A bag contains 4 white, 7 black and 5 red balls. 4 balls are drawn with replacement. What is the probability that at least two are white?


Answer:

Given:


Bag contains 4 white, 7 black and 5 red balls


It is told that four balls are drawn with replacement.


We need to find the probability that at least two balls drawn are white.


Let us find the individual probabilities to draw a ball


⇒ P(W) = P(drawing a white ball)




⇒ P(O) = P(drawing balls other than white)




The possible cases of drawing balls are as shown below:


i. Two whites and two others – 6 cases to draw


ii. Three whites and 1 others – 4 cases to draw


iii. Four whites – 1 case to draw


Since drawing balls are independent events so, the probabilities multiply each other.






∴ The required probability is .



Question 19.

Three cards are drawn with replacement from a well shuffled pack of 52 cards. Find the probability that the cards are a king, a queen and a jack.


Answer:

Given that three cards are drawn from a well-shuffled deck of 52 cards with replacement.


We know that there will 4 kings, 4 queens, 4 jacks present in a deck.


Let us find the probability of drawing the cards.


⇒ P(K) = P(Drawing King from 52 cards deck)




⇒ P(Q) = P(Drawing Queen from 52 cards deck)




⇒ P(J) = P(Drawing Jack from 52 cards deck)




We need to find the probability of drawing one king, one queen, one jack


There will be 6 cases or sequences for drawing these three cards


⇒ P(DKQJ) = P(Drawing King, queen and jack)


Since drawing cards are independent their probabilities multiply each other,




∴ The required probability is .



Question 20.

A bag contains 4 red and 5 black balls, a second bag contains 3 red and 7 black balls. One ball is drawn at random from each bag, find the probability that the (i) balls are of different colours (ii) balls are of the same colour.


Answer:

Given:


⇒ Bag A contains 4 red balls and 5 black balls


⇒ Bag B contains 3 red balls and 7 black balls


It is told that one ball is drawn is drawn from is each bag.


We need to find the probability that the balls are of same colour.


Let us find the Probability of drawing each colour ball from the bag.


⇒ P(B1) = P(drawing black ball from bag A)





⇒ P(R1) = P(drawing red ball from bag A)





⇒ P(B2) = P(drawing black ball from bag B)





⇒ P(R2) = P(drawing red ball from bag B)





We need to find the probability of drawing the different colour balls from two bags


⇒ P(DRB) = P(drawing one red ball and one Black ball)


⇒ P(DRB) = P(drawing black balls from bag A and red ball from bag B) + P(drawing black balls from bag B and red ball from bag A)


Since drawing a ball is independent for each bag, the probabilities multiply each other.





.


We need to find the probability of drawing the same colour balls from two bags


⇒ P(S) = P(drawing two balls of same colours) = P(drawing black balls from each bag) + (P(drawing white balls from each bag)


Since drawing a ball is independent for each bag, the probabilities multiply each other.





.


∴ The required probabilities are .



Question 21.

A can hit a target 3 times in 6 shots, B:2 times in 6 shots and C:4 times in 4 shots. They fix a volley. What is the probability that at least 2 shots hit?


Answer:

Given:


A hits a target 3 times out of 6 shots


B hits a target 2 times out of 6 shots


C hits a target 4 times out of 4 shots


⇒ P(TA) = P(A hits target)



⇒ P(TB) = P(B hits target)



⇒ P(TC) = P(C hits target)



⇒ P(NA) = P(A doesn’t hits target)




⇒ P(NB) = P(B doesn’t hits target)




⇒ P(NC) = P(C doesn’t hits target)



.


It is told that the target is to be hit by at least two shots. This is only possible when at least two of the Persons hits the target.


⇒ P(M) = P(TATBNC) + P(TANBTC) + P(NATBTc) + P(TATBTC)


Since hitting by a person is independent the probabilities will multiply each other.


⇒ P(M) = (P(TA)P(TB)P(NC)) + (P(TA)P(NB)P(TC)) + (P(NA)P(TB)P(Tc)) + (P(TA)P(TB)P(TC))




.



∴ The required probability is .



Question 22.

The probability of student A passing an examination is 2/9 and of student B passing is 5/9. Assuming the two events: ‘A passes’, ‘B passes’ as independent, find the probability of : (i) only A passing the examination (ii) only one of them passing the examination.


Answer:

Given that,


⇒ P(AP) = P(A passing an examination)



⇒ P(AN) = P(A Not passing an examination)




⇒ P(BP) = P(B passing an examination)



⇒ P(BN) = P(B Not passing an examination)




We need to find probability that:


i. Only A passing the examination


ii. Only one of them passing the examination


⇒ P(SA) = P(Only A passing the examination)


This happens only in the case B must fail


Since passing examination is an independent event their probabilities multiply each other





⇒ P(Sone) = P(Only one of them passed the examination)


⇒ P(Sone) = P(only A passed the examination) + P(only B passed the examination)


Since passing examination is an independent event their probabilities multiply each other






∴ The required probabilities are .



Question 23.

There are three urns A, B and C. Urn A contains 4 red balls and 3 black balls. Urn B contains 5 red balls and 4 black balls. Urn C contains 4 red and 4 black balls. One ball is drawn from each of these urns. What is the probability that 3 balls drawn consist of 2 red balls and a black ball?


Answer:

Given:


⇒ Urn A contains 4 red balls and 3 black balls


⇒ Urn B contains 5 red balls and 4 black balls


⇒ Urn C contains 4 red balls and 4 black balls


It is told that one ball is drawn is drawn from is each urn.


We need to find the probability that two balls are red and other ball is black.


Let us find the Probability of drawing each colour ball from the Urn.


⇒ P(B1) = P(drawing black ball from Urn A)





⇒ P(R1) = P(drawing Red ball from urn A)





⇒ P(B2) = P(drawing black ball from Urn B)





⇒ P(R2) = P(drawing Red ball from urn B)





⇒ P(B3) = P(drawing black ball from Urn C)





⇒ P(R3) = P(drawing Red ball from urn C)





We need to find the probability of 2 red balls and 1 black ball from three bags


⇒ P(S) = P(drawing two red ball and one Black ball)


⇒ P(S) = P(drawing black balls from bag A red ball from bag B and Red ball from bag C) + P(drawing black balls from bag B red ball from bag A and Red ball from bag C) + P(drawing black balls from bag C red ball from bag A and Red ball from bag B)


Since drawing a ball is independent for each bag, the probabilities multiply each other.






.


∴ The required probability is .



Question 24.

X is taking up subjects – Mathematics, Physics and Chemistry in the examination. His probabilities of getting grade A in these subjects are 0.2, 0.3 and 0.5 respectively. Find the probability that he gets

i. Grade A in all subjects

ii. Grade A in no subjects

iii. Grade A in two subjects


Answer:

Given:


⇒ P(MA) = P(getting A in mathematics)


⇒ P(MA) = 0.2


⇒ P(MN) = P(not getting A in mathematics)


⇒ P(MN) = 1-0.2


⇒ P(MN) = 0.8


⇒ P(PA) = P(getting A in physics)


⇒ P(PA) = 0.3


⇒ P(PN) = P(not getting A in physics)


⇒ P(PN) = 1-0.7


⇒ P(PN) = 0.3


⇒ P(CA) = P(getting A in Chemistry)


⇒ P(CA) = 0.5


⇒ P(CN) = P(not getting A in chemistry)


⇒ P(CN) = 1-0.5


⇒ P(CN) = 0.5


We need to find the probability that:


i. X gets A in all subjects


ii. X gets A in no subjects


iii. X gets A in two subjects


⇒ P(Xall) = P(getting A in all subjects)


Since getting A in different subjects is an independent event, their probabilities multiply each other


⇒ P(Xall) = (P(MA)P(PA)P(CA))


⇒ P(Xall) = 0.2×0.3×0.5


⇒ P(Xall) = 0.03


⇒ P(Xnone) = P(getting A in no subjects)


Since getting A in different subjects is an independent event, their probabilities multiply each other


⇒ P(Xnone) = (P(MN)P(PN)P(CN))


⇒ P(Xnone) = 0.8×0.7×0.5


⇒ P(Xnone) = 0.28


⇒ P(Xtwo) = P(getting A in any two subjects)


Since getting A in different subjects is an independent event, their probabilities multiply each other


⇒ P(Xtwo) = (P(MA)P(PA)P(CN)) + (P(MA)P(PN)P(CA)) + (P(MN)P(PA)P(CA))


⇒ P(Xtwo) = (0.2×0.3×0.5) + (0.2×0.7×0.5) + (0.8×0.3×0.5)


⇒ P(Xtwo) = 0.03 + 0.07 + 0.12


⇒ P(Xtwo) = 0.22


∴ The required probabilities are 0.03, 0.28, 0.22.



Question 25.

A and B take turns in throwing two dice, the first to throw 9 being awarded the prize. Show that their chance of winning are in the ration 9:8.


Answer:

Given that A and B throws two dice.


The first who throw 9 awarded a prize.


The possibilities of getting 9 on throwing two dice are:



⇒ P(S9) = P(getting sum 9)




⇒ P(SN) = P(not getting sum 9)




Let us assume A starts the game, A wins the game only when he gets 9 while throwing dice in 1st,3rd,5th,…… times


Here the probability of getting sum 9 on throwing a dice is same for both the players A and B


Since throwing a dice is an independent event, their probabilities multiply each other


⇒ P(Awins) = P(S9) + P(SN)P(SN)P(S9) + P(SN)P(SN)P(SN)P(SN)P(S9) + ……………




The series in the brackets resembles the Infinite geometric series.


We know that sum of a infinite geometric series with first term ‘a’ and common ratio ‘o’ is .






⇒ P(Bwins) = 1-P(Awins)





⇒ P(Awins):P(Bwins) = 9:8


∴ Thus proved



Question 26.

A, B and C in order toss a coin. The one to throw a head wins. What are their respective chances of winning assuming that the game may continue indefinitely?


Answer:

Given that A, B and C toss a coin until one of them gets a head to win the game.


Let us find the probability of getting the head.


⇒ P(AH) = P(A getting a head on tossing a coin)



⇒ P(AN) = P(A not getting head on tossing a coin)



⇒ P(BH) = P(B getting a head on tossing a coin)



⇒ P(BN) = P(B not getting head on tossing a coin)



⇒ P(CH) = P(C getting a head on tossing a coin)



⇒ P(CN) = P(C not getting head on tossing a coin)



It is told that A starts the game.


A tosses in1st,4th,7th,…… tosses.


This can be shown as follows:


⇒ P(WA) = P(A wins the game)


⇒ P(WA) = P(AH) + P(ANBNCNAH) + P(ANBNCNANBNCNAH) + …………………


Since tossing a coin by each person is an independent event, the probabilities multiply each other.


⇒ P(WA) = (P(AH)) + (P(AN)P(BN)P(CN)P(AH)) + ………………




The terms in the bracket resembles the infinite geometric series sequence:


We know that the sum of a Infinite geometric series with first term ‘a’ and common ratio ‘r’ is






B tosses in2nd,5th,8th,…… tosses.


This can be shown as follows:


⇒ P(WB) = P(B wins the game)


⇒ P(WB) = P(ANBH) + P(ANBNCNANBH) + P(ANBNCNANBNCNANBH) + …………………


Since tossing a coin by each person is an independent event, the probabilities multiply each other.


⇒ P(WB) = (P(AN)P(BH)) + (P(AN)P(BN)P(CN)P(AN)P(BH)) + ………………




The terms in the bracket resembles the infinite geometric series sequence:


We know that the sum of a Infinite geometric series with first term ‘a’ and common ratio ‘r’ is






⇒ P(WC) = P(winning of C)


⇒ P(WC) = 1-P(WA)-P(WB)




∴ The chances of winning of A,B and C are .



Question 27.

Three persons A, B, C throw a die in succession till one gets a ‘six’ and wins the game. Find their respective probabilities of winning.


Answer:

Given that A,B and C throws a die.


The first who throw 6 wins the game.


⇒ P(S6) = P(getting 6)



⇒ P(SN) = P(not getting 6)




Let us assume A starts the game, A wins the game only when he gets 6 while throwing dice in 1st,4th,7th,…… times


Here the probability of getting sum 6 on throwing a dice is same for the players A, B and C


Since throwing a dice is an independent event, their probabilities multiply each other


⇒ P(Awins) = P(S9) + P(SN)P(SN)P(SN)P(S9) + P(SN)P(SN)P(SN)P(SN)P(SN)P(SN)P(S9) + ……………




The series in the brackets resembles the Infinite geometric series.


We know that sum of a infinite geometric series with first term ‘a’ and common ratio ‘o’ is .






B wins the game only when he gets 6 while throwing dice in 2nd,5th,8th,…… times and others doesn’t get 6.


Since throwing a dice is an independent event, their probabilities multiply each other


⇒ P(Bwins) = (P(SN)P(S9)) + (P(SN)P(SN)P(SN)P(SN)P(S9)) + (P(SN)P(SN)P(SN)P(SN)P(SN)P(SN)P(SN)P(S9)) + ……………




The series in the brackets resembles the Infinite geometric series.


We know that sum of a infinite geometric series with first term ‘a’ and common ratio ‘o’ is .






⇒ P(Cwins) = 1-P(Awins)-P(Bwins)




∴ The probabilities of winning of A, B and C is .



Question 28.

A and B take turns in throwing two dice, the first to throw 10 being awarded the prize, show that if A has the first throw, their chance of winning are in the ratio 12:11.


Answer:

Given that A and B throws two dice.


The first who throw 10 awarded a prize.


The possibilities of getting 10 on throwing two dice are:



⇒ P(S10) = P(getting sum 10)




⇒ P(SN) = P(not getting sum 10)




Let us assume A starts the game, A wins the game only when he gets 10 while throwing dice in 1st,3rd,5th,…… times


Here the probability of getting sum 10 on throwing a dice is same for both the players A and B


Since throwing a dice is an independent event, their probabilities multiply each other


⇒ P(Awins) = P(S10) + P(SN)P(SN)P(S10) + P(SN)P(SN)P(SN)P(SN)P(S10) + ……………




The series in the brackets resembles the Infinite geometric series.


We know that sum of a infinite geometric series with first term ‘a’ and common ratio ‘o’ is .






⇒ P(Bwins) = 1-P(Awins)





⇒ P(Awins):P(Bwins) = 12:11


∴ Thus proved



Question 29.

There are 3 red and 5 black balls in bag ‘A’; and 2 red and 3 black balls in bag ‘B’. One ball is drawn from bag ‘A’ and two from bag ‘B’. Find the probability that out of the 3 balls drawn one is red and 2 are black.


Answer:

Given:


⇒ Bag A contains 3 red balls and 5 black balls


⇒ Bag B contains 2 red balls and 3 black balls


It is told that one ball is drawn from bag A and two balls from bag B.


We need to find the probability that one ball is red and other two are black.


Let us find the Probability of drawing each colour ball from the bag.


⇒ P(B1) = P(drawing black ball from bag A)





⇒ P(R1) = P(drawing Red ball from bag A)





⇒ P(B21) = P(drawing black ball from bag B in 1st draw)





⇒ P(R21) = P(drawing Red ball from bag B in 1st draw)





⇒ P(B22) = P(drawing black ball from bag B in 2nd draw after drawing red ball)





⇒ P(R22) = P(drawing Red ball from bag B in 2nd draw after drawing Black ball)





⇒ P(B221) = P(drawing black ball from bag B in 2nd draw after drawing black ball)





We need to find the probability of drawing a red and two black balls from two bags


⇒ P(S) = P(drawing one red ball and two Black balls)


⇒ P(S) = P(drawing red ball from bag A and black balls from bag B) + P(drawing black ball from bag A and red and black balls from bag B)


Since drawing a ball is independent for each bag, the probabilities multiply each other.





.


∴ The required probability is .



Question 30.

Fatima and John appear in an interview for two vacancies for the same post. The probability of Fatima’s selection is 1/7 and that of John’s selection is 1/5. What is the probability that

i. Both of them are selected

ii. Only one of them will be selected

iii. None of them will be selected


Answer:

Given that,


⇒ P(FS) = P(Fatima’s selection)



⇒ P(FN) = P(Not selecting Fatima)




⇒ P(JS) = P(John’s selection)



⇒ P(JN) = P(Not selecting John)




We need to find:


i. Both of them will be selected


ii. Only one of them will be selected


iii. None of them will be selected


⇒ P(Sboth) = P(Both of them are selected)


Since selection of each person is an independent event their probabilities multiply each other





⇒ P(Sone) = P(Only one of them is selected)


⇒ P(Sone) = P(only Fatima is selected) + P(only John is selected)


Since selection of each person is an independent event their probabilities multiply each other





⇒ P(Snone) = P(None of them are selected)


Since selection of each person is an independent event their probabilities multiply each other





∴ The required probabilities are .



Question 31.

A bag contains 8 marbles of which 3 are blue and 5 are red. One marble is drawn at random, its colour is noted and the marble is replaced in the bag. A marble is again drawn from the bag and its colour is noted. Find the probability that the marble will be

i. Blue followed by red

ii. Blue and red in any order

iii. Of the same colour


Answer:

Given:


Bag contains 3 blue and 5 red marbles


It is told that two marbles are drawn with replacement


Let us find the probability of drawing each marble from bag


⇒ P(B) = P(Drawing a Blue Marble)




⇒ P(R) = P(Drawing a Red Marble)




We need to find the probability that the marbles drawn:


i. Blue followed by red


ii. Blue and red in any order


iii. Of the same colour


⇒ P(SBR) = P(drawing Blue marble followed by Red)


Since drawing a marble is an independent event, the probabilities multiply each other.





⇒ P(Sany) = P(drawing Blue and red marble in any order)


⇒ P(Sany) = P(drawing Blue marble followed by red) + P(drawing Red marble followed by Blue)


Since drawing a marble is an independent event, the probabilities multiply each other.





.


⇒ P(S) = P(drawing two marbles of same colour)


⇒ P(S) = P(drawing black balls from each bag) + (P(drawing white balls from each bag)


Since drawing a ball is independent for each bag, the probabilities multiply each other.





.


∴ The required probabilities are .



Question 32.

An urn contains 7 red and 4 blue balls. Two balls are drawn at random with replacement. Find the probability of getting.

i. 2 red balls

ii. 2 blue balls

iii. One red and one blue ball


Answer:

Given:


Urn contains 7 red and 4 blue balls


It is told that two balls are drawn with replacement


Let us find the probability of drawing each marble from bag


⇒ P(B) = P(Drawing a Blue Ball)





⇒ P(R) = P(Drawing a Red Ball)





We need to find the probability that the balls drawn:


i. Both in red colour


ii. Both in blue colour


iii. One red and one blue


⇒ P(SRR) = P(drawing Two red colour balls)


Since drawing a ball is an independent event, the probabilities multiply each other.





⇒ P(SBB) = P(drawing two blue colour balls)


Since drawing a ball is independent for each bag, the probabilities multiply each other.





⇒ P(Sany) = P(drawing Blue and red ball in any order)


⇒ P(Sany) = P(drawing Blue ball followed by red) + P(drawing Red ball followed by Blue)


Since drawing a ball is an independent event, the probabilities multiply each other.





.


∴ The required probabilities are .



Question 33.

A card is drawn from a deck of 52 cards. The outcome is noted, the card is replaced and the deck reshuffled. Another card is then drawn from the deck.

i. What is the probability that both the cards are of the same suit?

ii. What is the probability that the first card is an ace and the second card is a red queen?


Answer:

Given that two cards are drawn from the deck with replacement.


We know that there will four suits in a deck and each suit contains 13 cards namely Spades, Hearts, Diamonds, Clubs.


Also, there will total of 4 aces and 2 red queens and 2 black queens


Let us find the probability required:


⇒ P(Dsame) = P(For selecting a card from a single suit out of 52 cards)




This probability will be same for all the suits.



⇒ P(Dace) = P(Drawing an ace)




⇒ P(DredQ) = P(Drawing a red queen)




We need to find the probability of getting:


i. Both cards from same deck


ii. First an ace and second a red queen


⇒ P(Ssame) = P(getting both cards from same deck)


We may get two cards any of the four decks. So, each deck’s probability is taken into consideration.


⇒ P(Ssame) = P(both cards from spade) + P(both cards from hearts) + P(both cards from diamond) + P(both cards from club)


Since drawing a card is an independent event, their probabilities multiply each other.


⇒ P(Ssame) = (P(DS)P(DS)) + (P(DH)P(DH)) + (P(DD)P(DD)) + (P(DC)P(DC))






⇒ P(SAR) = P(getting an ace first and followed red queen)


Since drawing a card is an independent event, their probabilities multiply each other.


⇒ P(SAR) = (P(DA)P(DredQ))




∴ The required probabilities are .



Question 34.

Out of 100 students, two sections of 40 and 60 are formed. If you and your friend are among 100 students, what is the probability that: (i) you both enter the same section? (ii) you both enter the different section?


Answer:

Given:


Section A contains 40 students


Section B contains 60 students


It is told that you and your friend are among 100 students


We need to find:


i. You both enter the same section


ii. You both enter the different section


⇒ P(Ssame) = P(You and your friend enter same section)


This happens only in the case if you and your friend belongs to either section A or section B.


⇒ P(Ssame) = P(Both belongs to section A) + P(Both belongs to section B)








⇒ P(Sdiff) = P(You and your friend enter different section)


This happens only in the case if you belongs to A and your friend belongs to section B.


⇒ P(Sdiff) = 1-P(Both belongs to same section)


⇒ P(Sdiff) = 1-P(Ssame)




∴ The required probabilities are .



Question 35.

In a hockey match, both teams A and B scored same number of goals up to the end of the game, so to decide the winner, the referee asked both the captains throw a die alternately and decide that the team, whose captain gets a first six, will be declared the winner. If the captain of team A was asked to start, find their respective probabilities of winning the match and state whether the decision of the referee was fair or not.


Answer:

Given that teams A and B scored same number of goals.


It is asked captains of A and B to throw a die.


The first who throw 6 awarded a prize.


⇒ P(S6) = P(getting 6)



⇒ P(SN) = P(not getting 6)




It is given A starts the game, A wins the game only when he gets 6 while throwing die in 1st,3rd,5th,…… times


Here the probability of getting 6 on throwing a die is same for both the players A and B


Since throwing a die is an independent event, their probabilities multiply each other


⇒ P(Awins) = P(S6) + P(SN)P(SN)P(S6) + P(SN)P(SN)P(SN)P(SN)P(S6) + ……………




The series in the brackets resembles the Infinite geometric series.


We know that sum of a infinite geometric series with first term ‘a’ and common ratio ‘o’ is .






⇒ P(Bwins) = 1-P(Awins)




Since the probabilities if winnings of A and B are not equal, the decision of the referee is not fair



Question 36.

A and B throw a pair of die alternately. A wins the game if he gets a total of 7 and B wins the game if he gets a total of 10. If A starts the game, then find the probability that B wins.


Answer:

Given that A and B throw a pair of die alternatively.


It is told that A wins if he gets sum of 7 and B wins if he gets sum of 10.


Let us find the probability of getting sum 7 and 10.


The possibilities of getting 7 on throwing a pair of die:


{(1,6),(2,5),(3,4),(4,3),(5,2),(6,1)} = 6 cases


The possibilities of getting 10 on throwing a pair of die:


{(4,6),(5,5),(6,4)} = 3 cases


⇒ P(S7) = P(getting sum 7)





⇒ P(SN7) = P(not getting sum 7)


⇒ P(SN7) = 1-P(S7)




⇒ P(S10) = P(getting sum 10)





⇒ P(SN10) = P(not getting sum 10)


⇒ P(SN10) = 1-P(S10)




It is told that A starts the game.


We need to find the probability that B wins the games.


B wins the game only when A losses in1st,3rd,5th,…… throws.


This can be shown as follows:


⇒ P(BWINS) = P(B wins the game)


⇒ P(BWins) = P(SN7S10) + P(SN7SN10SN7S10) + P(SN7SN10SN7SN10SN7S10) + …………………


Since throwing a pair of die by each person is an independent event, the probabilities multiply each other.


⇒ P(Bwins) = (P(SN7)P(S10)) + (P(SN7)P(SN10)P(SN7)P(S10)) + ………………




The terms in the bracket resembles the infinite geometric series sequence:


We know that the sum of a Infinite geometric series with first term ‘a’ and common ratio ‘r’ is





∴ The required probability is .




Exercise 31.6
Question 1.

A bag A contains 5 white and 6 black balls. Another bag B contains 4 white and 3 black balls. A ball is transferred from bag A to the bag B, and then a ball is taken out of the second bag. Find the probability of this ball being black.


Answer:

Given:


Bag A contains 5 white and 6 black balls.


Bag B contains 4 white and 3 black balls.


A ball is transferred from bag A to bag B, and then a ball is drawn from bag B.


There are two mutually exclusive ways to draw a black ball from bag B –


a. A white ball is transferred from bag A to bag B, and then, a black ball is drawn from bag B


b. A black ball is transferred from bag A to bag B, and then, a black ball is drawn from bag B


Let E1 be the event that white ball is drawn from bag A and E2 be the event that black ball is drawn from bag A.


Now, we have





We also have





Let E3 denote the event that black ball is drawn from bag B.


Hence, we have





We also have





Using the theorem of total probability, we get


P(E3) = P(E1)P(E3|E1) + P(E2)P(E3|E2)





Thus, the probability of the drawn ball being black is.



Question 2.

A purse contains 2 silver and 4 copper coins. A second purse contains 4 silver and 3 copper coins. If a coin is pulled at random from one of the two purses, what is the probability that it is a silver coin?


Answer:

Given:


First purse contains 2 silver and 4 copper coins.


Second purse contains 4 silver and 3 copper coins.


A coin is pulled a random from one of the two purses.


There are two mutually exclusive ways to pull a silver coin from one of the two purses –


a. The first purse is selected, and then, a silver coin is pulled from the first purse


b. The second purse is selected, and then, a silver coin is pulled from the second purse


Let E1 be the event that the first purse is selected and E2 be the event that the second purse is selected.


Since there are only two purses and each purse has an equal probability of being selected, we have



Let E3 denote the event that a silver coin is pulled.


Hence, we have






We also have





Using the theorem of total probability, we get


P(E3) = P(E1)P(E3|E1) + P(E2)P(E3|E2)






Thus, the probability of pulling a silver coin is.



Question 3.

One bag contains 4 yellow and 5 red balls. Another bag contains 6 yellow and 3 red balls. A ball is transferred from the first bag to the second bag, and then a ball is drawn from the second bag. Find the probability that ball drawn is yellow.


Answer:

Given:


The bag I contains 4 yellow and 5 red balls.


Bag II contains 6 yellow and 3 red balls.


A ball is transferred from bag I to bag II and then a ball is drawn from bag II.


There are two mutually exclusive ways to draw a yellow ball from bag II –


a. A yellow ball is transferred from the bag I to bag II, and then, a yellow ball is drawn from bag II


b. A red ball is transferred from the bag I to bag II, and then, a yellow ball is drawn from bag II


Let E1 be the event that yellow ball is drawn from the bag I and E2 be the event that red ball is drawn from the bag I.


Now, we have





We also have





Let E3 denote the event that yellow ball is drawn from bag II.


Hence, we have





We also have





Using the theorem of total probability, we get


P(E3) = P(E1)P(E3|E1) + P(E2)P(E3|E2)






Thus, the probability of the drawn ball being yellow is.



Question 4.

A bag contains 3 white and 2 black balls, and another bag contains 2 white and 4 black balls. One bag is chosen at random. From the selected bag, one ball is drawn. Find the probability that the ball drawn is white.


Answer:

Given:


The bag I contains 3 white and 2 black balls.


Bag II contains 2 white and 4 black balls.


A bag is chosen, and a ball is drawn from it.


There are two mutually exclusive ways to draw a white ball from one of the two bags –


a. The bag I is selected, and then, a white ball is drawn from the bag I


b. Bag II is selected, and then, a white ball is drawn from bag II


Let E1 be the event that bag I is selected and E2 be the event that bag II is selected.


Since there are only two bags and each bag has an equal probability of being selected, we have



Let E3 denote the event that a white ball is drawn.


Hence, we have





We also have






Using the theorem of total probability, we get


P(E3) = P(E1)P(E3|E1) + P(E2)P(E3|E2)







Thus, the probability of the drawn ball being white is.



Question 5.

The contents of three bags I, II and III are as follows:

Bag I: 1 white, 2 black and 3 red balls

Bag II: 2 white, 1 black and 1 red ball

Bag III: 4 white, 5 black and 3 red balls

A bag is chosen at random and two balls are drawn. What is the probability that the balls drawn are white and red?


Answer:

Given:


Bag I contains 1 white, 2 black and 3 red balls


Bag II contains 2 white, 1 black and 1 red ball


Bag III contains 4 white, 5 black and 3 red balls


A bag is chosen and two balls are drawn from it.


There are three mutually exclusive ways to draw a white and a red ball from one of the three bags –


a. Bag I is selected, and then, a white and a red ball are drawn from bag I


b. Bag II is selected, and then, a white and a red ball are drawn from bag II


c. Bag III is selected, and then, a white and a red ball are drawn from bag III


Let E1 be the event that bag I is selected, E2 be the event that bag II is selected and E3 be the event that bag III is selected.


Since there are only three bags and each bag has an equal probability of being selected, we have



Let E4 denote the event that a white and a red ball are drawn.


Hence, we have







We also have







Similarly, we also have







Using the theorem of total probability, we get


P(E4) = P(E1)P(E4|E1) + P(E2)P(E4|E2) + P(E3)P(E4|E3)








Thus, the probability of the drawn balls being white and red is.



Question 6.

An unbiased coin is tossed. If the result is a head, a pair of unbiased dice is rolled and the sum of the numbers obtained is noted. If the result is a tail, a card from a well shuffled pack of eleven cards numbered 2,3,4,…,12 is picked and the number on the card is noted. What is the probability that the noted number is either 7 or 8?


Answer:

Given:


An unbiased coin is tossed.


If heads occurs, a pair of dice is rolled and sum is noted.


If tails occurs, a number from 2, 3, 4,…, 12 is noted.


There are two mutually exclusive ways to note a number, which is either 7 or 8 –


a. Toss result is a head, and then, sum of the obtained numbers on rolling two dice is 7 or 8


b. Toss result is a tail, and then, a card numbered 7 or 8 is picked from the pack


Let E1 be the event that the result of the toss is a head and E2 be the event that the result of the toss is a tail.


Since there are only two outcomes for a coin toss and each outcome has an equal probability of occurring, we have



Let E3 denote the event that the noted number is 7 or 8.


Hence, we have



When two dice are rolled, we obtain a sum of 7 when the outcomes are (1, 6), (2, 5), (3, 4), (4, 3), (5, 2) and (6, 1).


We obtain a sum of 8 when the outcomes are (2, 6), (3, 5), (4, 4), (5, 3) and (6, 2).


Hence, 11 ways out of 36 give the sum 7 or 8.



We also have




Using the theorem of total probability, we get


P(E3) = P(E1)P(E3|E1) + P(E2)P(E3|E2)






Thus, the probability of the noted number being 7 or 8 is.



Question 7.

A factory has two machines A and B. Past records show that the machine A produced 60% of the items of output and machine B produce 40% of the items. Further 2% of the items produced by machine A were defective and 1% produced by machine B were defective. If an item is drawn at random, what is the probability that it is defective?


Answer:

Given:


Machine A produced 60% of the total items.


Machine B produced 40% of the total items.


2% of the items produced by machine A were defective.


1% of the items produced by machine B were defective.


There are two mutually exclusive ways to draw a defective item produced by one of the two machines –


a. Item was produced by machine A, and then, the item is defective


b. Item was produced by machine B, and then, the item is defective


Let E1 be the event that the item was produced by machine A and E2 be the event that the item was produced by machine B.


As 60% of the total items were produced by machine A, we have




Similarly, as 40% of the total items were produced by machine B, we have




Let E3 denote the event that the item is defective.


Hence, we have



2% of the items produced by machine A are defective.



We also have



1% of the items produced by machine A are defective.



Using the theorem of total probability, we get


P(E3) = P(E1)P(E3|E1) + P(E2)P(E3|E2)






Thus, the probability of the item being defective is.



Question 8.

The bag A contains 8 white and 7 black balls while the bag B contains 5 white and 4 black balls. One balls is randomly picked up from the bag A and mixed up with the balls in bag B. Then a ball is randomly drawn out from it. Find the probability that ball drawn is white.


Answer:

Given:


Bag A contains 8 white and 7 black balls.


Bag B contains 5 white and 4 black balls.


A ball is transferred from bag A to bag B and then a ball is drawn from bag B.


There are two mutually exclusive ways to draw a white ball from bag B –


a. A white ball is transferred from bag A to bag B, and then, a white ball is drawn from bag B


b. A black ball is transferred from bag A to bag B, and then, a white ball is drawn from bag B


Let E1 be the event that white ball is drawn from bag A and E2 be the event that black ball is drawn from bag A.


Now, we have





We also have





Let E3 denote the event that white ball is drawn from bag B.


Hence, we have





We also have





Using the theorem of total probability, we get


P(E3) = P(E1)P(E3|E1) + P(E2)P(E3|E2)





Thus, the probability of the drawn ball being white is.



Question 9.

A bag contains 4 white and 5 black balls and another bag contains 3 white and 4 black balls. A ball is taken out from the first bag and without seeing its colour is put in the second bag. A ball is taken out form the latter. Find the probability that the ball drawn is white.


Answer:

Given:


Bag I contains 4 white and 5 black balls.


Bag II contains 3 white and 4 black balls.


A ball is transferred from bag I to bag II and then a ball is drawn from bag II.


There are two mutually exclusive ways to draw a white ball from bag II –


a. A white ball is transferred from bag I to bag II, and then, a white ball is drawn from bag II


b. A black ball is transferred from bag I to bag II, and then, a white ball is drawn from bag II


Let E1 be the event that white ball is drawn from bag I and E2 be the event that black ball is drawn from bag I.


Now, we have





We also have





Let E3 denote the event that white ball is drawn from bag II.


Hence, we have





We also have





Using the theorem of total probability, we get


P(E3) = P(E1)P(E3|E1) + P(E2)P(E3|E2)





Thus, the probability of the drawn ball being white is.



Question 10.

One bag contains 4 white and 5 black balls. Another bag contains 6 white and 7 black balls. A ball is transferred from first bag to the second bag and then a ball is drawn from the second bag. Find the probability that the ball drawn is white.


Answer:

Given:


Bag I contains 4 white and 5 black balls.


Bag II contains 6 white and 7 black balls.


A ball is transferred from bag I to bag II and then a ball is drawn from bag II.


There are two mutually exclusive ways to draw a white ball from bag II –


a. A white ball is transferred from bag I to bag II, and then, a white ball is drawn from bag II


b. A black ball is transferred from bag I to bag II, and then, a white ball is drawn from bag II


Let E1 be the event that white ball is drawn from bag I and E2 be the event that black ball is drawn from bag I.


Now, we have





We also have





Let E3 denote the event that white ball is drawn from bag II.


Hence, we have





We also have





Using the theorem of total probability, we get


P(E3) = P(E1)P(E3|E1) + P(E2)P(E3|E2)






Thus, the probability of the drawn ball being white is.



Question 11.

An urn contains 10 white and 3 black balls. Another urn contains 3 white and 5 black balls. Two balls are drawn from first urn and put into the second urn and then a ball is drawn from the latter. Find the probability that it is a white ball.


Answer:

Given:


Urn I contains 10 white and 3 black balls


Urn II contains 3 white and 5 black balls


Two balls are transferred from urn I to urn II and then a ball is drawn from urn II.


There are three mutually exclusive ways to draw a white ball from urn II –


a. Two white balls are transferred from urn I to urn II, and then, a white ball is drawn from urn II


b. Two black balls are transferred from urn I to urn II, and then, a white ball is drawn from urn II


c. A white and a black ball are transferred from urn I to urn II, and then, a white ball is drawn from urn II


Let E1 be the event that two white balls are drawn from urn I, E2 be the event that two black balls are drawn from urn I and E3 be the event that a white and a black ball are drawn from urn I.


Now, we have







We also have







Similarly, we have







Let E4 denote the event that a white is drawn.


Hence, we have





We also have





Similarly, we also have





Using the theorem of total probability, we get


P(E4) = P(E1)P(E4|E1) + P(E2)P(E4|E2) + P(E3)P(E4|E3)






Thus, the probability of the drawn ball being white is.



Question 12.

A bag contains 6 red and 8 black balls and another bag contains 8 red and 6 black balls. A ball is drawn from the first bag and without noticing its colour is put in the second bag. A ball is drawn from the second bag. Find the probability that the ball drawn is red in colour.


Answer:

Given:


Bag I contains 6 red and 8 black balls.


Bag II contains 8 red and 6 black balls.


A ball is transferred from bag I to bag II and then a ball is drawn from bag II.


There are two mutually exclusive ways to draw a red ball from bag II –


a. A red ball is transferred from bag I to bag II, and then, a red ball is drawn from bag II


b. A black ball is transferred from bag I to bag II, and then, a red ball is drawn from bag II


Let E1 be the event that red ball is drawn from bag I and E2 be the event that black ball is drawn from bag I.


Now, we have






We also have






Let E3 denote the event that red ball is drawn from bag II.


Hence, we have





We also have





Using the theorem of total probability, we get


P(E3) = P(E1)P(E3|E1) + P(E2)P(E3|E2)





Thus, the probability of the drawn ball being red is.



Question 13.

Three machines E1,E2,E3 in a certain factory produce 50%, 25% and 25% respectively, of the total daily output of electric bulbs. It is known that 4% of the tubes produced one each of machines E1 and E2 are defective, and that 5% of those produced on E3 are defective. If one tube is picked up at random from a day’s production, calculate the probability that it is defective.


Answer:

Given:


Machine E1 produces 50% of the total output.


Machine E2 produces 25% of the total output.


Machine E3 produces 25% of the total output.


4% of the tubes produced by machine E1 are defective.


4% of the tubes produced by machine E2 are defective.


5% of the tubes produced by machine E3 are defective.


There are three mutually exclusive ways to pick up a defective tube produced by one of the three machines –


a. Tube was produced by machine E1, and then, the tube is defective


b. Tube was produced by machine E2, and then, the tube is defective


c. Tube was produced by machine E3, and then, the tube is defective


Let X1 be the event that the tube is produced by machine E1, X2 be the event that the tube is produced by machine E2 and X3 be the event that the tube is produced by machine E3.


As 50% of the total output is produced by machine E1, we have




Similarly, as each of machines E2 and E3 produces 25% of the total tubes, we have




Let X4 denote the event that the tube is defective.


Hence, we have



4% of the tubes produced by machine E1 are defective.



Similarly,


We also have



5% of the tubes produced by machine E3 are defective.



Using the theorem of total probability, we get


P(X4) = P(X1)P(X4|X1) + P(X2)P(X4|X2) + P(X3)P(X4|X3)








Thus, the probability of the tube being defective is.




Exercise 31.7
Question 1.

The contents of the urns I,II,III are as follows:

Urn I: 1 white, 2 black and 3 red balls

Urn II: 2 white, 1 black and 1 red balls

Urn III: 4 white, 5 black and 3 red balls.

One urn is chosen at random and two balls are drawn. They happen to be white and red. What is the probability that they come from Urns I,II,III?


Answer:

Given:


Urn I has 1 white, 2 black and 3 red balls


Urn II has 2 white, 1 black and 1 red balls


Urn III has 4 white, 5 black and 3 red balls


Let us assume U1, U2, U3 and A be the events as follows:


U1 = choosing Urn I


U2 = choosing Urn II


U3 = choosing Urn III


A = choosing 1 white and 1 red ball from urn


We know that each urn is most likely to choose. So, probability of choosing a urn will be same for every Urn.





The Probability of choosing balls from each Urn differs from Urn to Urn and the probabilities are as follows:


⇒ P(A|U1) = P(Choosing required balls from Urn 1)






⇒ P(A|U2) = P(Choosing required balls from Urn 2)






⇒ P(A|U3) = P(Choosing required balls from Urn 3)






Now we find


P(U1|A) = P(The chosen balls are from Urn1)


P(U2|A) = P(The chosen balls are from Urn2)


P(U3|A) = P(The chosen balls are from Urn3)


Using Baye’s theorem:

















∴ The required probabilities are .



Question 2.

A bag A contains 2 white and 3 red balls and a bag B contains 4 white and a bag B contains 4 white and 5 red balls. One ball is drawn at random from one of the bags and it is found to be red. Find the probability that it was drawn from bag B.


Answer:

Given:


Bag A has 2 white and 3 red balls


Bag B has 4 white and 5 red balls


Let us assume B1, B2, B3 and A be the events as follows:


B1 = choosing Bag I


B2 = choosing Bag II


A = choosing red ball from Bag


We know that each Bag is most likely to choose. So, probability of choosing a bag will be same for every bag.




The Probability of choosing balls from each Bag differs from bag to bag and the probabilities are as follows:


⇒ P(A|B1) = P(Choosing red ball from Bag 1)





⇒ P(A|B2) = P(Choosing red ball from Bag 2)





Now we find


P(B2|A) = P(The chosen ball is from bag2)


Using Baye’s theorem:







∴ The required probabilities are .



Question 3.

Three urns contains 2 white and 3 black balls; 3 white and 2 black balls; 4 white and 1 black balls respectively. One ball is drawn from an urn chosen at random and it was found to be white. Find the probability that it was drawn from the first urn.


Answer:

Given:


Urn I has 2 white and 3 black balls


Urn II has 3 white and 2 black balls


Urn III has 4 white, 1 black red balls


Let us assume U1, U2, U3 and A be the events as follows:


U1 = choosing Urn I


U2 = choosing Urn II


U3 = choosing Urn III


A = choosing 1 white ball from an urn


We know that each urn is most likely to choose. So, probability of choosing a urn will be same for every Urn.





The Probability of choosing balls from each Urn differs from Urn to Urn and the probabilities are as follows:


⇒ P(A|U1) = P(Choosing white ball from Urn 1)





⇒ P(A|U2) = P(Choosing white ball from Urn 2)





⇒ P(A|U3) = P(Choosing required balls from Urn 3)





Now we find


P(U1|A) = P(The chosen balls are from Urn1)


Using Baye’s theorem:







∴ The required probabilities is .



Question 4.

The contents of the three urns are as follows:

Urn I contains 7 white and 3 black balls

Urn II contains 4 white and 6 black balls

Urn III contains 2 white and 8 black balls.

One of these urns are chosen at random probabilities 0.20,0.60 and 0.20 respectively. From the chosen urn two balls are drawn without replacement. If both these balls are white, what is the probability that these came from urn 3?


Answer:

Given:


Urn I has 7 white and 3 black balls


Urn II has 4 white and 6 black balls


Urn III has 2 white and 8 black balls


Let us assume U1, U2, U3 and A be the events as follows:


U1 = choosing Urn I


U2 = choosing Urn II


U3 = choosing Urn III


A = choosing 1 white and 1 red ball from urn


From the problem,





The Probability of choosing balls from each Urn differs from Urn to Urn and the probabilities are as follows:


⇒ P(A|U1) = P(Choosing required balls from Urn 1)






⇒ P(A|U2) = P(Choosing required balls from Urn 2)






⇒ P(A|U3) = P(Choosing required balls from Urn 3)






Now we find


P(U3|A) = P(The chosen balls are from Urn3)


Using Baye’s theorem:







∴ The required probabilities is .



Question 5.

Suppose a girl thrown a die. If she gets 1 or 2, she tosses a coin three times and notes the number of tails. If she gets 3,4,5 or 6, she tosses a coin once and notes whether a ‘head’ or ‘tail’ is obtained. If she obtained exactly one ‘tail’, what is the probability that she threw 3,4,5 or 6 with the die?


Answer:

Given:


Let us assume D1, D2 and A be the events as follows:


D1 = Throwing die and getting 1 or 2


D2 = Throwing die and getting 3,4,5 or 6


A = getting exactly one tail






⇒ P(A|D1) = P(getting 1 tail on getting 1 or 2 from die)





⇒ P(A|D2) = P(Getting 1 tail on getting 3,4,5 or 6 from die)





Now we find


P(D2|A) = P(The tail we get here come after getting 3,4,5 or 6 from die)


Using Baye’s theorem:







∴ The required probability is .



Question 6.

Two groups are competing for the positions of the Board of Directors of corporation. The probabilities that the first and the second groups will win are 0.6 and 0.4 respectively. Further, if the first group wins, the probability of introducing a product is 0.7 and the corresponding probability is 0.3 if the second group wins. Find the probability that the new product introduced was by the second group.


Answer:

Let us assume U1, U2 and A be the events as follows:


U1 = First team wins


U2 = Second team wins


A = Introduction of project


From the problem ir is given that,




⇒ P(A|U1) = P(introduction of project by team 1) = 0.7


⇒ P(A|U2) = P(introduction of project by team 2) = 0.3


Now we find


P(U2|A) = P(Project introduced by team 2)


Using Baye’s theorem:







∴ The required probability is .



Question 7.

Suppose 5 men out of 100 and 25 women out of 1000 are good orators. An orator is chosen at random. Find the probability that a male person is selected. Assume that there are equal number of men and women.


Answer:

Let us assume U1, U2 and A be the events as follows:


U1 = choosing men


U2 = choosing women


A = choosing orator


From the problems it is clear that men and women are equal in number. So, probability of choosing a men and women will be same.




From the problem:


⇒ P(A|U1) = P(Choosing orator from men) =


⇒ P(A|U2) = P(Choosing orator from women) =


Now we find


P(U1|A) = P(The chosen orator is men)


Using Baye’s theorem:







∴ The required probability is .



Question 8.

A letter is known to have come either from LONDON or CLIFTON. On the envelope just two consecutive letters are visible. What is the probability that the letter has come from

i. LONDON

ii. CLIFTON


Answer:

Let us assume U1, U2 and A be the events as follows:


U1 = Letter comes from LONDON


U2 = Letter comes from CLIFTON


A = Event of getting two visible consecutive letters are ON


We know that letter comes from each place is most likely to occur. So, probability of getting letter will be same from each place.




Now,


⇒ P(A|U1) = P(visible two letters ON from london )




⇒ P(A|U2) = P(visible two letters ON from CLIFTON)




Now we find


P(U1|A) = P(The envelope comes from LONDON)


P(U2|A) = P(The envelope comes from CLIFTON)


Using Baye’s theorem:












The required probabilities are .



Question 9.

In a class 5% of the boys and 10% of the girls have an IQ of more than 150. In this class, 60% of the students are boys. If a student is selected at random and found to have an IQ of more than 150, find the probability that the student is a boy.


Answer:

Let us assume U1, U2 and A be the events as follows:


U1 = choosing boy


U2 = choosing girl


A = choosing a student with IQ more than 150


From the problem:




⇒ P(A|U1) = P(Boy whose IQ is more than 150)



⇒ P(A|U2) = P(Girl whose IQ is more than 150)



Now we find


P(U1|A) = P(The choosen student whose IQ is more than 150 is a boy)


Using Baye’s theorem:







∴ The required probabilities is .



Question 10.

A factory has three machines X,Y and Z producing 1000, 2000 and 3000 bolts per day respectively. The machine X produces 1% defectives bolts, Y produces 1.5% and Z produces 2% defective bolts. At the end of a day, a bolt is drawn at random and is found to be defective. What is the probability that this defective bolt has been produced by machine X?


Answer:

Given:


Factory X has produced 1000 bolts per day


Factory Y has produced 2000 bolts per day


Factory Z has produced 3000 bolts per day.


Let us assume U1, U2, U3 and A be the events as follows:


U1 = choosing factory X


U2 = choosing factory Y


U3 = choosing factory Z


A = choosing a defective bolt





From the problem:


⇒ P(A|U1) = P(Choosing defective bolt from factory X)



⇒ P(A|U2) = P(Choosing defective bolt from factory Y)



⇒ P(A|U3) = P(Choosing defective bolt from factory Z)



Now we find


P(U1|A) = P(The chosen bolt is from factory X)


Using Baye’s theorem:







∴ The required probability is .



Question 11.

An insurance company insured 3000 scooters, 4000 cars and 5000 trucks. The probabilities of the accident involving a scooter, a car and a truck are 0.02, 0.03 and 0.04 respectively. One of the insured vehicles meet with and accident. Find the probability that it is a (i) scooter (ii) car (iii) truck.


Answer:

Given:


Company has 3000 scooters, 4000 cars and 5000 trucks.


Let us assume U1, U2, U3 and A be the events as follows:


U1 = choosing scooter


U2 = choosing car


U3 = choosing truck


A = Involving in accident





From the problem:


⇒ P(A|U1) = P(scooter involving in accident)



⇒ P(A|U2) = P(car involving in a accident)



⇒ P(A|U3) = P(truck involving in accident)



Now we find


P(U1|A) = P(The vehicle which met with accident is a scooter)


P(U2|A) = P(The vehicle which met with accident is a car)


P(U3|A) = P(The vehicle which met with accident is a truck)


Using Baye’s theorem:

















∴ The required probabilities are .



Question 12.

Suppose we have four boxes A,B,C,D containing coloured marbles as given below:



One of the boxes has been selected at random and a single marble is drawn from it. If the marble is red, what is the probability that it was drawn form box A? box B? box C?


Answer:

Let us assume U1, U2, U3, U4 and A be the events as follows:


U1 = choosing Box A


U2 = choosing Box B


U3 = choosing Box C


U4 = choosing Box D


A = choosing red marble from box


We know that each box is most likely to choose. So, probability of choosing a box will be same for every Urn.






The Probability of choosing marble from each box differs from box to box and the probabilities are as follows:


⇒ P(A|U1) = P(Choosing red marble from Box A)





⇒ P(A|U2) = P(Choosing red marble from Box B)





⇒ P(A|U3) = P(Choosing red marble from Box C)





Since there is no red marbles in Box D, the probability of choosing red marble is 0.


i.e, P(A|U4) = 0


Now we find


P(U1|A) = P(The chosen red marble is from Box A)


P(U2|A) = P(The chosen red marble is from Box B)


P(U3|A) = P(The chosen red marble is from Box C)


Using Baye’s theorem:














∴ The required probabilities are .


Question 13.

A manufacturer has three machine operators A,B and C. The first operator A produces 1% defective items, whereas the other two operators B and C produce 5% and 7% defective items respectively. A is on the job for 50% of the time, B on the job for 30% of the time and C on the job for 20% of the time. A defective item is produced. What is the probability that it was produced by A?


Answer:

Let us assume U1, U2, U3 and A be the events as follows:


U1 = choosing Operator A


U2 = choosing Operator B


U3 = choosing Operator C


A = Producing defective item


From the problem:





⇒ P(A|U1) = P(Producing defective item by operator A)



⇒ P(A|U2) = P(Producing defective item by operator B)



⇒ P(A|U3) = P(Producing defective item by operator C)



Now we find


P(U1|A) = P(The defective item is produced by operator A)


Using Baye’s theorem:






∴ The required probability is .



Question 14.

An item is manufactured by three machines A,B and C. Out of the total number of items manufactured during a specified period, 50% are manufacture on machine A, 30% on B and 20% on C. 2% of the items produced on A and 2% of items produced on B are defective and 3% of these produced on C are defective. All the items stored at one godown. One item is drawn at random and is found to be defective. What is the probability that it was manufactured on machine A?


Answer:

Let us assume U1, U2, U3 and A be the events as follows:


U1 = choosing Machine A


U2 = choosing Machine B


U3 = choosing Machine C


A = manufacturing defective item





⇒ P(A|U1) = P(Manufacturing defective item from machine A)



⇒ P(A|U2) = P(Manufacturing defective item from machine B)



⇒ P(A|U3) = P(Manufacturing defective item from machine C)



Now we find


P(U1|A) = P(The defective item is manufactured from machine A)


Using Baye’s theorem:







∴ The required probability is .



Question 15.

There are three coins. One is two - headed coin (having head on both faces), another is biased coin that comes up heads 75% of the times the third is also a biased coin that comes up tail 40% of the times. One of the three coins is chosen at random and tossed, and it shows heads. What is the probability that it was the two - headed coin?


Answer:

Given:


Coin 1 has heads on both sides


Coin 2 and 3 are biased coins


Let us assume U1, U2, U3 and A be the events as follows:


U1 = choosing coin 1


U2 = choosing coin 2


U3 = choosing coin 3


A = getting head on tossing the coin


We know that each coin is most likely to choose. So, probability of choosing a coin will be same for every coin.





From the problem:


⇒ P(A|U1) = P(getting head on tossing coin 1)



⇒ P(A|U2) = P(getting head on tossing coin 2)



⇒ P(A|U3) = P(getting head on tossing coin 3)



Now we find


P(U1|A) = P(The head we get after is from tossing coin 1)


Using Baye’s theorem:







∴ The required probability is .



Question 16.

In a factory, machine A produces 30% of the total output, machine B produces 25% and the machine C produces the remaining output. If defective items produced by machines A, B, C are 1%,1.2%, 2% respectively. Three machines working together produce 10000 items in a day. An item is drawn at random form a day’s output and found to be defective. Find the probability that it was produced by machine B?


Answer:

Let us assume U1, U2, U3 and A be the events as follows:


U1 = choosing Machine A


U2 = choosing Machine B


U3 = choosing Machine C


A = Producing a defective output


From the problem:





⇒ P(A|U1) = P(Producing defective output from Machine A)



⇒ P(A|U2) = P(Producing defective output from Machine B)



⇒ P(A|U3) = P(Producing defective output from Machine C)



Now we find


P(U2|A) = P(The found defective item is produced by Machine B)


Using Baye’s theorem:







∴ The required probability is .



Question 17.

A company has two plants to manufacture bicycles. The first plant manufactures 60% of the bicycles and the second plant 40%. Out of that 80% of the bicycles are rated of standard quality at the first plant and 90% of standard quality at the second plant. A bicycle is picked up at random and found to be standard quality. Find the probability that it comes from the second plant.


Answer:

Let us assume U1, U2 and A be the events as follows:


U1 = Choosing first plant to manufacture bicycles


U2 = choosing second plant to manufacture bicycles


A = Picking standard quality cycle


From the Problem




⇒ P(A|U1) = P(Picking standard quality cycle from first plant)



⇒ P(A|U2) = P(Picking standard quality cycle from second plant)



Now we find


P(U2|A) = P(The chosen standard quality cycle is from second plant)


Using Baye’s theorem:







∴ The required probability is .



Question 18.

Three urns A,B and C contain 6 red and 4 white; 2 red and 6 white; and 1 red and 5 white balls respectively. An urn is chosen at random and a ball is drawn. If the ball drawn is found to be red, find the probability that the ball was drawn from urn A.


Answer:

Given:


Urn A has 6 red and 4 white balls


Urn B has 2 red and 6 white balls


Urn C has 1 red and 5 white balls


Let us assume U1, U2, U3 and A be the events as follows:


U1 = choosing Urn A


U2 = choosing Urn B


U3 = choosing Urn C


A = choosing red ball from urn


We know that each urn is most likely to choose. So, probability of choosing a urn will be same for every Urn.





The Probability of choosing balls from each Urn differs from Urn to Urn and the probabilities are as follows:


⇒ P(A|U1) = P(Choosing red ball from Urn A)





⇒ P(A|U2) = P(Choosing red ball from Urn B)





⇒ P(A|U3) = P(Choosing red ball from Urn C)





Now we find


P(U1|A) = P(The red ball is from Urn A)


Using Baye’s theorem:







∴ The required probability is .



Question 19.

In a group of 400 people, 160 are smokers and non - vegetarian, 100 are smokers and vegetarian and the remaining are non - smokers and vegetarian. The probabilities of getting a special chest disease are 35%, 20% and 10% respectively. A person is chosen from the group at random and is found to be suffering from the disease. What is the probability that the selected person is a smoker and non - vegetarian?


Answer:

Given:


From 400 people


Smokers and non - vegetarian are 160


Smokers and vegetarian are 100


Non - smokers and vegetarian are 140


Let us assume U1, U2, U3 and A be the events as follows:


U1 = choosing Smokers and non - vegetarian


U2 = choosing Smokers and vegetarian


U3 = choosing Non - smokers and vegetarian


A = getting special chest disease


From the problem





⇒ P(A|U1) = P(Smoker and non - vegetarian getting Chest disease)



⇒ P(A|U2) = P(Smoker and vegetarian getting chest disease)



⇒ P(A|U3) = P(Non - smoker and vegetarian getting chest disease)



Now we find


P(U1|A) = P(The selected chest diseased person is a smoker and non - vegetarian)


Using Baye’s theorem:







∴ The required probability is .



Question 20.

A factory has three machines A,B and C, which produce 100,200 and 300 items of a particular type daily. The machines produce 2%, 3% and 5% defective items respectively. One day when the production was over, and item was picked up randomly and it was found to be defective. Find the probability that it was produced by machine A.


Answer:

Given:


Machine A produce 100 items


Machine B produce 200 items


Machine C produce 300 items


Let us assume U1, U2, U3 and A be the events as follows:


U1 = choosing Machine A


U2 = choosing Machine B


U3 = choosing Machine C


A = Producing defective item by machine





From the problem


⇒ P(A|U1) = P(Producing defective item by Machine A)



⇒ P(A|U2) = P(Producing defective item by Machine B)



⇒ P(A|U3) = P(Producing defective item by Machine C)



Now we find


P(U1|A) = P(The chosen defective item is produced by machine A)


Using Baye’s theorem:







∴ The required probability is .



Question 21.

A bag contains 1 white and 6 red balls, and a second bag contains 4 white and 3 red balls. One of the bags is picked up at random and a ball is randomly drawn from it, and is found to be white in colour. Find the probability that the drawn ball was from the first bag.


Answer:

Given:


Bag I has 1 white and 6 red balls


Bag II has 4 white and 3 red balls


Let us assume U1, U2 and A be the events as follows:


U1 = choosing Bag I


U2 = choosing Bag II


A = choosing white ball from urn


We know that each bag is most likely to choose. So, probability of choosing a bag will be same for every bag.




The Probability of choosing ball from each Bag differs from Bag to Bag and the probabilities are as follows:


⇒ P(A|U1) = P(Choosing white ball from Bag I)





⇒ P(A|U2) = P(Choosing white ball from Bag II)





Now we find


P(U1|A) = P(The chosen ball is from Bag I)


Using Baye’s theorem:







∴ The required probabilities are .



Question 22.

In a certain college, 4% of boys and 1% of girls are taller than 1.75 metres. Further more, 60% of the students in the college are girls. A student is found to be taller than 1.75 metres. Find the probability that the selected students is girl.


Answer:

Let us assume U1, U2 and A be the events as follows:


U1 = choosing Boy


U2 = choosing Girl


A = choosing student who is taller than 1.75 metres.


From the problem




⇒ P(A|U1) = P(Choosing boy who is taller than 1.75 metres)



⇒ P(A|U2) = P(Choosing girl taller than 1.75 metres)



Now we find


P(U2|A) = P(The chosen student is a girl taller than 1.75 metres)


Using Baye’s theorem:







∴ The required probability is .



Question 23.

For A,B and C the chances of being selected as the manager of a firm are in the ration 4:1:2 respectively. The respective probabilities for them to introduce a radical change in marketing strategy are 0.3, 0.8 and 0.5. If the change does take place, find the probability that it is due to the appointment of B or C.


Answer:

Let us assume U1, U2, U3 and A be the events as follows:


U1 = choosing Firm A


U2 = choosing Firm B


U3 = choosing Firm C


A = A change takes place


From the problem





⇒ P(A|U1) = P(Change occurs due to firm A)



⇒ P(A|U2) = P(Change occurs due to Firm B)



⇒ P(A|U3) = P(Change occurs due to firm C)



Now we find


P(U2|A) + P(U3|A) = P(The change occurred due to appointing B or C)


Using Baye’s theorem:







∴ The required probability is .



Question 24.

Three persons A,B and C apply for a job of Manager in a private company. Chances of their selection (A,B and C) are in the ratio 1:2:4. The probabilities that A,B and C can introduce changes to improve profits of the company are 0.8, 0.5 and 0.3 respectively. If the changes do not take place, find the probability that it is due to the appointment of C.


Answer:

Let us assume U1, U2, U3 and A be the events as follows:


U1 = choosing Person A


U2 = choosing Person B


U3 = choosing Person C


A = Improve in profit occurs


From the problem





⇒ P(A|U1) = P(Change occurs due to person A)



⇒ P(A|U2) = P(Change occurs due to person B)



⇒ P(A|U3) = P(Change occurs due to person C)



Now we find


1 - P(U3|A) = P(The change does not occurred due to appointing C)


Using Baye’s theorem:







∴ The required probability is .



Question 25.

An insurance company insured 2000 scooters and 3000 motorcycles. The probability of an accident involving a scooter is 0.01 and that of a motorcycle is 0.02. An insured vehicle met with an accident. Find the probability that the accidented vehicle was a motorcycle.


Answer:

Given:


Company insured 2000 scooters and 3000 motorcycles


Let us assume U1, U2 and A be the events as follows:


U1 = choosing Scooter


U2 = choosing Motorcycle


A = accident involving vehicle


From the problem




⇒ P(A|U1) = P(accident involving scooter)



⇒ P(A|U2) = P(accident involving motorcycles)



Now we find


P(U2|A) = P(The vehicle involved in an accident is motorcycle)


Using Baye’s theorem:







∴ The required probability is .



Question 26.

Of the students in a college, it is known that 60% reside in a hostel and 40% do not reseide in hostel. Previous year results report that 30% of students residing in hostel attain A grade and 20% of ones not residing in hostel attain A grade in their annual examination. At the end of the year, one students is chosen at random from the college and he has an A grade. What is the probability that the selected student is a hosteler?


Answer:

Let us assume U1, U2, U3 and A be the events as follows:


U1 = Student residing in Hostel


U2 = Student not residing in hostel


A = Attaining A grade


From the problem




⇒ P(A|U1) = P(student in hostel attains A grade)



⇒ P(A|U2) = P(Student not in hostel attains A grade)



Now we find


P(U1|A) = P(The student who got A grade is a hosteller)


Using Baye’s theorem:







∴ The required probabilities are .



Question 27.

There are three coins. One is two headed coin, another is a biased coin that comes up heads 75% of the time and third is an unbiased coin. One of the three coins is choosen at random and tossed, it shows heads, what is the probability that it was the two headed coin?


Answer:

Given:


Coin 1 is two heads, coin2 is biased and coin 3 is unbiased


Let us assume U1, U2, U3 and A be the events as follows:


U1 = choosing coin 1


U2 = choosing coin 2


U3 = choosing coin 3


A = getting heads


We know that each coin is most likely to choose. So, probability of choosing a coin will be same for every coin.





From the problem


⇒ P(A|U1) = P(getting heads on tossing coin 1)



⇒ P(A|U2) = P(getting heads on tossing coin 2)



⇒ P(A|U3) = P(getting heads on tossing coin 3)



Now we find


P(U1|A) = P(The coin tossed to get head is Coin 1)


Using Baye’s theorem:







∴ The required probability is .



Question 28.

Assume that the chances of a patient having a heart attack is 40%. It is also assumed that meditation and yoga course reduces the risk of heart attack by 30% and prescription of certain drug reduces its chances by 25%. At a time a patient can choose any one of the two options with equal probabilities. It is given that after going through one of the two options and patient selected at random suffers a heart attack. Find the probability that the patient followed a course of meditation and yoga?


Answer:

Let us assume U1, U2 and A be the events as follows:


U1 = Selecting meditation or yoga


U2 = selecting Drugs


A = Getting heart attack


From the problem,




⇒ P(A) = 0.4


⇒ P(A|U1) = P(getting heart attack even after practicing yoga or meditation)




⇒ P(A|U2) = P(getting heart attack even after using drugs)




Now we find


P(U1|A) = P(The patient got heart attack even after practicing yoga or meditation)


Using Baye’s theorem:







∴ The required probability is .



Question 29.

Coloured balls are distributed in four boxes as shown in the following table:



A box is selected at random and then a ball is randomly drawn from the selected box. The colour of the ball is black, what is the probability that ball drawn is from the box III.


Answer:

Given:


Box I has 3 Black, 4 White, 5 Red, 6 Blue balls


Box II has 2 Black, 2 White, 2 Red, 2 Blue balls


Box III has 1 Black, 2 White, 3 Red, 1 Blue balls


Box IV has 4 Black, 3 White, 1 Red, 5 Blue balls


Let us assume U1, U2, U3, U4 and A be the events as follows:


U1 = choosing Box I


U2 = choosing Box II


U3 = choosing Box III


U4 = choosing Box IV


A = choosing Black ball from box


We know that each urn is most likely to choose. So, probability of choosing a urn will be same for every Urn.






The Probability of choosing balls from each box differs from box to box and the probabilities are as follows:


⇒ P(A|U1) = P(Choosing black ball from Box I)





⇒ P(A|U2) = P(Choosing black ball from Box II)






⇒ P(A|U3) = P(Choosing black ball from Box III)





⇒ P(A|U3) = P(Choosing black ball from Box III)





Now we find


P(U3|A) = P(The black ball is from BallIII)


Using Baye’s theorem:







∴ The required probability is .



Question 30.

If a machine is correctly set up it produces 90% acceptable items. If it is incorrectly set up it produces only 40% acceptable items. Past experience shows that 80% of the setups are correctly done. If after a certain set up, the machine produces 2 acceptable items, find the probability that the machine is correctly set up.


Answer:

Let us assume U1, U2 and A be the events as follows:


U1 = Machine is correctly set up


U2 = Machine is incorrectly set up


A = produce two acceptable items


From the problem




⇒ P(A|U1) = P(producing 2 acceptable items if machine is correctly set up)




⇒ P(A|U2) = P(producing 2 acceptable items if machine is not correctly set up)




Now we find


P(U1|A) = P(Machine is correctly set up for producing 2 acceptable items)


Using Baye’s theorem:







∴ The required probability is .



Question 31.

Bag A contains 3 red and 5 black balls, white bag B contains 4 red and 4 black balls. Two balls are transferred at random from bag A to bag B and then a ball is drawn from bag B at random. If the ball drawn from bag B is found to be red, find the probability that two red balls were transferred from bag A to bag B.


Answer:

Given:


Bag A contains 3 red and 5 black balls


Bag B contains 4 red and 4 black balls


It is told that two balls are transferred from bag A to bag B, the possible cases (events) will be as follows:


(i) U1 = Transferring 2 red balls from bag A to bag B


(ii) U2 = Transferring 1 red ball and 1 black ball from bag A to bag B


(iii) U3 = Transferring 2 black balls from bag A to bag B


Let us assume the event A as follows:


⇒ A = Drawing red ball from bag B


Now,


⇒ P(U1) = P(transferring 2 red balls from bag A to bag B)





⇒ P(U2) = P(transferring 1 red and 1 black ball from bag A to bag B)





⇒ P(U3) = P(transferring 2 black balls from bag A to bag B)





⇒ P(A|U1) = P(drawing red ball after transferring 2 red balls from bag A to bag B)




⇒ P(A|U2) = P(drawing red ball after transferring 1 red and 1 black ball from bag A to bag B)




⇒ P(A|U3) = P(drawing red ball after transferring 2 black balls from bag A to bag B)




We need to find


⇒ P(U1|A) = P(red ball is drawn after transferring two red balls from bag A to bag B)


Using Baye’s theorem,






∴ The required probability is .



Question 32.

By examining the chest X - ray, probability that T.B is detected when a person is actually suffering is 0.99. The probability that the doctor diagnoses incorrectly that a person has T.B. on the basis of X - ray is 0.001. In a certain city 1 in 1000 persons suffers from T.B. A person is selected at random is diagnosed to have T.B. What is the chance that he actually has T.B.?


Answer:

Let us assume U1, U2 and A be the events as follows:


U1 = Person having T.B


U2 = Person not having T.B


A = Diagnosing T.B disease


From the problem:




⇒ P(A|U1) = P(diagnosing T.B disease for the person who actually having T.B)



⇒ P(A|U2) = P(diagnosing T.B disease for the person who don’t actually have T.B)



Now we find


P(U1|A) = P(The person has T.B and diagnosed T.B)


Using Baye’s theorem:







∴ The required probability is .



Question 33.

A test for detection of a particular disease is not fool proof. The test will correctly detect the disease 90% of the time, but will incorrectly detect the disease 1% of the time. For a large population of which an estimated 0.2% have the disease, a person is selected at random, given the test, and told that he has the disease. What are the chances that the person actually have the disease?


Answer:

Let us assume U1, U2 and A be the events as follows:


U1 = Person actually has a disease


U2 = Person doesn’t has a disease


A = detection of disease


From the problem




⇒ P(A|U1) = P(Test correctly detected)



⇒ P(A|U2) = P(Test incorrectly detected)



Now we find


P(U1|A) = P(The person actually has the disease and correctly detected)


Using Baye’s theorem:







∴ The required probability is .



Question 34.

Let d1, d2, d3 be three mutually exclusive diseases. Let S be the set of observable symptoms of these diseases. A doctor has the following information from a random sample of 5000 patients: 1800 had disease d1, 2100 has disease d2 and the others had disease d3. 1500 patients with disease d1, 1200 patients with disease d2 and 900 patients with disease d3 showed the symptom. Which of the diseases is the patient most likely to have?


Answer:

Given:


1800 patients had disease d1


2100 patients had disease d2


1100 patients had disease d3


Let us assume U1, U2, U3 and A be the events as follows:


U1 = Person with disease d1


U2 = Person with disease d2


U3 = Person with disease d3


A = Showing the symptom S


Now,





⇒ P(A|U1) = P(patient who has disease d1 showed the symptom S)




⇒ P(A|U2) = P(patient who has disease d2 showed the symptom S)




⇒ P(A|U3) = P(patient who has disease d3 showed the symptom S)




Now we find


P(U1|A) = P(The patient who showed symptom S has disease d1)


P(U2|A) = P(The patient who showed symptom S has disease d2)


P(U3|A) = P(The patient who showed symptom S has disease d3)


Using Baye’s theorem:

















Since the probability of P(U1|A) is larger than other disease’s probability. The patient mostly likely to have the disease d1.



Question 35.

A is known to speak truth 3 times out of 5 times. He throws a die and reports that it is one. Find the probability that it is actually one.


Answer:

Let us assume U1, U2 and A be the events as follows:


U1 = Getting 1 on throwing a die


U2 = Getting other than 1 on throwing a die


A = Reporting 1 after throwing the die


From the problem,




⇒ P(A|U1) = P(Reporting 1 on actually getting 1 on throwing a die)


⇒ P(A|U1) = P(Telling the truth)



⇒ P(A|U2) = P(Reporting 1 but not getting 1 on throwing a die)


⇒ P(A|U2) = P(Not telling the truth)



Now we find


P(U1|A) = P(the die actually shows 1 given that man reports 1)


Using Baye’s theorem:






∴ The required probability is .



Question 36.

A speaks the truth 8 times out of 10 times. A die is tossed. He reports that it was 5. What is the probability that it was actually 5?


Answer:

Let us assume U1, U2 and A be the events as follows:


U1 = Getting 5 on throwing a die


U2 = Getting other than 5 on throwing a die


A = Reporting 5 after throwing the die


From the problem,




⇒ P(A|U1) = P(Reporting 5 on actually getting 5 on throwing a die)


⇒ P(A|U1) = P(Telling the truth)



⇒ P(A|U2) = P(Reporting 5 but not getting 5 on throwing a die)


⇒ P(A|U2) = P(Not telling the truth)



Now we find


P(U1|A) = P(the die actually shows 5 given that man reports 5)


Using Baye’s theorem:







∴ The required probability is .



Question 37.

In answering a question on a multiple choice test, a student either knows the answer or guesses. Let 3/4 be the probability that he knows the answer and 1/4 be the probability that he guesses. Assuming that a student who guesses at the answer will be correct with probability 1/4. What is the probability that a student knows the answer given that he answered it correctly?


Answer:

Let us assume U1, U2, and A be the events as follows:


U1 = Knowing the answer


U2 = Guessing the answer


A = Answering the question correctly


From the problem




⇒ P(A|U1) = P(Answering the question correctly by knowing the answer)


⇒ P(A│U1 ) = 1


⇒ P(A|U2) = P(Answering the question correctly by guessing the answer)



Now we find


P(U1|A) = P(The answer is known given that he answerd the question correctly)


Using Baye’s theorem:







∴ The required probability is.



Question 38.

A laboratory blood test is 99% effective in detecting a certain disease when its infection is present. However, the test also yields a false positive result for 0.5% of the healthy person tested (i.e if a healthy person is tested, then, with probability 0.005, the test will imply he has the disease). If 0.1% of the population actually has the disease, what is the probability that a person has the disease given that his test result is positive?


Answer:

Let us assume U1, U2, U3 and A be the events as follows:


U1 = Person has disease


U2 = Person doesn’t has disease


A = Blood test result is positive


From the problem,




⇒ P(A|U1) = P(Blood test results shows positive for the person with disease)



⇒ P(A|U2) = P(Blood test results shows positive for the person without disease)



Now we find


P(U1|A) = P(The Person has a disease given that the blood test results shows positive)


Using Baye’s theorem:








∴ The required probability is .



Question 39.

There are three categories of students in a class of 60 students: A: Very hardworking; B: Regular but not so hardworking; C: Careless and irregular 10 students are in category A, 30 in category B and rest in category C. It is found that the probability of students of category A, unable to get good marks in the final year examination is 0.002, of category B it is 0.02 and of category C, this probability is 0.20. A student selected at random was found to be one who could not get good marks in the examination. Find the probability that this student is of category C.


Answer:

Given:


10 students are in category A


30 students are in category B


20 students are in category C


Let us assume U1, U2, U3 and A be the events as follows:


U1 = Choosing student from category A


U2 = choosing student from category B


U3 = choosing student from category C


A = Not getting good marks in final examination


Now,





⇒ P(A|U1) = P(student not getting good marks from category A)



⇒ P(A|U2) = P(student not getting good marks from category B)



⇒ P(A|U3) = P(student not getting good marks from category C)



Now we find


P(U3|A) = P(The student is from category C given that he didn’t get good marks in final examination)


Using Baye’s theorem:







∴ The required probability is .




Very Short Answer
Question 1.

A four digit number is formed using the digits 1, 2, 3, 5 with no repetitions. Write the probability that the number is divisible by 5.


Answer:


Given, a four-digit number is formed with the digits 1, 2, 3, 5 with no repetitions. There are as many numbers as there are ways of filling in 4 vacant places.


The no. of ways in which the 4 places can be filled, by multiplication principle, is ways


For the Numbers divisible by 5,



the 4th place can be only filled by 1 way i.e., by 5, the 1st place can be filled by 3 ways (since repetition is not allowed), the second place by 2 ways and the third by 1 way.


total numbers divisible by


P (no. divisible by 5)



Question 2.

When three dice are thrown, write the probability of getting 4 or 5 on each of the dice simultaneously.


Answer:

When three dice are thrown, no. of events that can happen

P(getting 4 or 5 simultaneously)



Question 3.

Three digit numbers are formed with the digits 0, 2, 4, 6 and 8. Write the probability of forming a three digit number with the same digits.


Answer:

The probability of forming 3-digit no. with digits 0,2,4,6,8 is as follows,

Since it is a 3 digit no., first place cannot have 0 as it will make it a 2 digit no.


1st place can be filled in 4 ways, 2nd place in 5 ways and 3rd in 5 ways.


 total number


The three digit numbers with same digits are 222,444,666,888


P (forming three-digit no.)



Question 4.

A ordinary cube has four plane faces, one face marked 2 and another face marked 3, find the probability of getting a total of 7 in 5 throws.


Answer:

Total no. of ways= 65

A total of 7 in 5 throws can be obtained by getting two 2’s ,one 3 and two blanks.





Question 5.

Three numbers are chosen from 1 to 20. Find the probability that they are consecutive.


Answer:

Given, no from 1 to 20

Sample space=20!


Three no consecutively=18!×3!


Now, considering three numbers as a single digit, we get 18 nos and these can be arranged in ways





Question 6.

6 boys and 6 girls sit in a row at random. Find the probability that all the girls sit together.


Answer:

Given there are 6 boys and 6 girls

No of ways in which 6 boys and 6 girls sitting together in a row=7!


6 girls sitting arrangement=


required probability=




Question 7.

If A and B are two independent events such that P(A) = 0.3 and . Find P(B).


Answer:

Given A and B are independent events =>


Since A and B are independent, A and are also independent


)


By theorem,



0.8= 0.3+(1-P(B))-0.3(1-P(B))


0.8= 0.3+1-P(B)-0.3+0.3P(B)


0.8-1= -0.7P(B)


-0.2= -0.7P(B)





Question 8.

An unbiased die with face marked 1, 2, 3, 4, 5, 6 is rolled four times. Out of 4 face values obtained, find the probability that the minimum face value is not less than 2 and the maximum face value is not greater than 5.


Answer:

Favourable points are 2,3,4,5

Therefore, Probability=


Since the die is rolled 4 times


∴ Required probability




Question 9.

If A and B are two events write the expression for the probability of occurrence of exactly one of two events.


Answer:

If A and B are independent, then

P (exactly one of them occurs) = P(A) + P(B) — 2P(A)× P(B)



Question 10.

Write the probability that a number selected at random from the set of first 100 natural numbers is a cube.


Answer:

Let S = (1,2,3,………..,100)

Let A be the set of cubes from the set of first 100 natural numbers,


A = (1,8,27,64)


P (selected no. being cube)




=0.04



Question 11.

In a competition A, B and C are participating. The probability that A wins is twice that of B, the probability that B wins is twice that of C . Find the probability that A losses.


Answer:

Given P(A)=2 P(B)

P(B)=2 P(C)


P(A)=2(2P(C))


=4P(C)


By definition we know,


P(A) +P(B)+ P(C)=1


⇒ 4P(C) + 2P(C) +P(C) =1


⇒ 7P(C)=1


⇒ P(C)=


Therefore P(A) = 4 P(C)




P(A loses)




Question 12.

If A, B, C are mutually exclusive and exhaustive events associated to a random experiment, then write the value of P(A) + P(B) + P(C).


Answer:

Given


Also given


By formula we know,


Substituting the values we get


1 = P(A)+P(B)+P(C)


⇒ P(A)+P(B)+P(C) = 1



Question 13.

If two events A and B are such that , and , find


Answer:

Given

We know that


But, P(A)=1-0.3=0.7


Therefore,


Now, P(A∪B)=P(A)+P(B)-P(A∩B)


= 0.7 + 0.4-0.4


= 0.9


Therefore,


=1-0.9


=0.1




Question 14.

If A and B are two independent events, then write ) in terms of P(A) and P(B).


Answer:

Given, A and B are independent.

A and are also independent.



=P(A)(1-P(B))


=P(A)-P(A)P(B)



Question 15.

If P(A) = 0.3, P(B) = 0.6, P(B/A) = 0.5, find


Answer:

Given, P(A)=0.3,P(B)=0.6,P(




=0.5 X 0.3


=0.15


P(A∪B)=P(A)+P(B)-P(A∩B)


=0.3+0.6-0.15


=0.75



Question 16.

If A, B and C are independent events such that P(A) = P(B) = P(C) = p, then find the probability of occurrence of at least two of A, B and C.


Answer:

Given, P(A)=P(B)=P(C)=p

P(occurrence of at least two of A,B,C)=P(A)P(B)P(C ̅ )+P(A)P(B ̅ )P(C)+P(A ̅ )P(B)P(C)+P(A)P(B)P(C)


=p×p(1-p)+p×(1-p)p+(1-p)p × p + p × p × p


=3p2 (1-p)+p3


=3p2 -3p3 +p3


=3p2 -2p3



Question 17.

If A and B are independent events then write expression for P (exactly one of A, B occurs).


Answer:

Given A and B are independent events, then

P(exactly one of A,B occurs)



Question 18.

If A and B are independent events such that P(A) = p, p(B) = 2p and P (Exactly one of A and B occurs) = 5/9, find the value of p.


Answer:

Given P(A)=p and P(B)=2p

P(exactly one of A and B occurs)= P(A)P(+P(P(B)





⇒27p-36p2 -5=0


⇒36p2 -27p+5=0


⇒36p2 -12p-15p+5=0


⇒12p(3p-1)-5(3p-1)=0


⇒ (12p-5)(3p-1)=0


⇒12p-5=0



also, 3p-1=0



value of




Mcq
Question 1.

Mark the correct alternative in each of the following:

If one ball is drawn at random from each of three boxes containing 3 white and 1 black, 2 white and 2 black, 1 white and 3 black balls, then the probability that 2 white and 1 black balls will be drawn is

A. 13/32

B. 1/4

C. 1/32

D. 3/16


Answer:

There are 3 ways of drawing 2 white balls and 1 black ball from the three boxes, i.e.,

WWB,WBW,BWW


For box 1,



For box 2,



For box 3,







Question 2.

Mark the correct alternative in each of the following:

A and B draw two cards each, one after another, from a pack of well-shuffled pack of 52 cards. The probability that all the four cards drawn are of the same suit is

A.

B.

C.

D. none of these


Answer:

Total No. Of cards =52

No. Of cards for each suit = 13


Total types Of French suit = 4



For this conditional case of selecting all four same suit,it can be either hearts or spades or clubs or diamonds


So Total no. Of conditional case= no. of case of selecting all four cards of hearts suit +spades suit + clubs suit + diamonds suit.




2860


Total no. of case


=270725


So, probability of getting all four cards of the same suit



Question 3.

Mark the correct alternative in each of the following:

A and B are two events such that P(A) = 0.25 and P(B) = 0.50. The probability of both happening together is 0.14. The probability of both A and B not happening is

A. 0.39

B. 0.25

C. 0.11

D. none of these


Answer:

Given P(A∪B)=0.14,P(A)=0.25,P(B)=0.50

By addition theorem,


P(A∪B)=P(A)+P(B)-P(A∩B)


P(A∩B)=0.75-0.14=0.61


∴P(both A and B not happening)=1-P(A∩B)=1-0.61=0.39


Question 4.

Mark the correct alternative in each of the following:

The probabilities of a student getting I, II and III division in an examination are respectively. The probability that the student fails in the examination is

A.

B.

C.

D. none of these


Answer:

P(student fails)=P(student not getting 1st division)XP(student not getting 2nd division)XP(student not getting 3rd division)



Question 5.

Mark the correct alternative in each of the following:

India play two matches each with West Indies and Australia. In any match the probabilities of India getting 0, 1 and 2 points are 0.45, 0.05 and 0.50 respectively. Assuming that the outcomes are independent, the probability of India getting at least 7 points is

A. 0.0875

B. 1/16

C. 0.1125

D. none of these


Answer:

India wins = W = 2 pt

India loses = L = 0 pt


India draws = D = 1 pt


Total matches played by India = 4


This means that the total points needs to be 7 at least


Either India wins all 4 matches or India wins 3 and draws 1


the probability of WWWW = (0.5)*(0.5)*(0.5)*(0.5) = 0.0625


India wins 3 matches and draws 1: probability in that case = (0.5)*(0.5)*(0.5)*(0.05) = 0.00625


The 4 possible outcomes will be:


WWWD, WWDW, WDWW, DWWW


Total probability of India wins 3 matches and draws 1 = 4*(0.00625) = 0.025


P( total points have to be at least 7) = 0.0625 + 0.025 = 0.0875


Question 6.

Mark the correct alternative in each of the following:

Three faces of an ordinary dice are yellow, two faces are red and one face is blue. The dice is rolled 3 times. The probability that yellow red and blue face appear in the first second and third throws respectively, is

A. 1/36

B. 1/6

C. 1/30

D. none of these


Answer:

Probability of getting blue face is .


Probability of getting yellow face is .


Probability of getting red face is .


probability that yellow red and blue face appear in the first second and third throws is



Question 7.

Mark the correct alternative in each of the following:

The probability that a leap year will have 53 Friday 53 Saturdays is

A. 2/7

B. 3/7

C. 4/7

D. 1/7


Answer:

There are 366 days in a leap year.
52×7


=364 days.
So, there are 7 possible combinations for the last 2 days (Sunday, Monday) (Monday, Tuesday) ... (Saturday, Sunday). Of these 7, 3 give you either 53 Saturdays or Fridays.
So, ans is 3/7.


Question 8.

Mark the correct alternative in each of the following:

A person writes 4 letters and addresses 4 envelopes. If the letters are placed in the envelopes at random, then the probability that all letters are not placed in the right envelopes, is

A. 1/4

B. 11/24

C. 15/24

D. 23/24


Answer:

let P=all letters are dispatched in right letters

Therefore,





Question 9.

Mark the correct alternative in each of the following:

A speaks truth in 75% cases and B speaks truth in 80% cases. Probability that they contradict each other in a statement, is

A. 7/20

B. 13/20

C. 3/5

D. 2/5


Answer:

Let A = Event that A speaks the truth
B = Event that B speaks the truth
Then




P(A-lie)


P(B-lie)



Now,
A and B contradict each other =
[A lies and B true] or [B true and B lies]
= P(A).P(B-lie) + P(A-lie).P(B)




Question 10.

Mark the correct alternative in each of the following:

Three integers are chosen at random from the first 20 integers. The probability that their product is even is

A. 2/19

B. 3/29

C. 17/19

D. 4/19


Answer:

the product of 3 randomly chosen integers are even only if there are even nos.

P(even) = 1-P(odd)


P(odd)



P(even)


Question 11.

Mark the correct alternative in each of the following:

Out of 30 consecutive integers, 2 are chosen at random. The probability that their sum is odd, is

A. 14/29

B. 16/29

C. 15/29

D. 10/29


Answer:

. There are 15 odd numbers and 15 even numbers.
Sum is odd if you choose 1 odd and 1 even.
So, the probability of sum being odd is



Question 12.

Mark the correct alternative in each of the following:

A bag contains 5 black balls, 4 white balls and 3 red balls. If a ball is selected random wise, the probability that it is black or red ball is

A. 1/3

B. 1/4

C. 5/12

D. 2/3


Answer:

P(black or red)




Question 13.

Mark the correct alternative in each of the following:

Two dice are thrown simultaneously. The probability of getting a pair of aces is

A. 1/36

B. 1/3

C. 1/6

D. none of these


Answer:

for this experiment, out of the 36 sample point, the favorable one is (1,1)


Question 14.

Mark the correct alternative in each of the following:

An urn contains 9 balls two of which are red, three blue and four black. Three balls are drawn at random. The probability that they are of the same colour is

A. 5/84

B. 3/9

C. 3/7

D. 7/17


Answer:

P(same colour)



Question 15.

Mark the correct alternative in each of the following:

A coin is tossed three times. If events A and B are defined as A = Two heads come, B = Last should be head. Then, A and B are

A. independent

B. dependent

C. both

D. mutually exclusive


Answer:

since event A is two heads come and event B is last should be head, both are dependent.


Question 16.

Mark the correct alternative in each of the following:

Five persons entered the lift cabin on the ground floor of an 8 floor house. Suppose that each of them independently and with equal probability can leave the cabin at any floor beginning with the first, then the probability of all 5 persons leaving at different floors is

A. B.

C. D.


Answer:

P(all 5 persons leaving at diff. floors)=


Question 17.

Mark the correct alternative in each of the following:

A box contains 10 good articles and 6 with defects. One item is drawn at random. The probability that it is either good or has a defect is

A. 64/64

B. 49/64

C. 40/64

D. 24/64


Answer:

P(good articles)

P(defective)





Question 18.

Mark the correct alternative in each of the following:

A box contains 6 nails and 10 nuts. Half of the nails and half of the nuts are rusted. If one item is chosen at random, the probability that it is rusted or is a nail is

A. 3/16

B. 5/16

C. 11/16

D. 14/16


Answer:

Given half of nails and nuts are rusted.

Rusted nails=3


Rusted nuts=5


P(item rusted or nail)




Question 19.

Mark the correct alternative in each of the following:

A bag contains 5 brown and 4 white socks. A man pulls out two socks. The probability that these are of the same colour is

A. 5/108

B. 18/108

C. 30/108

D. 48/108


Answer:

2 brown socks from 5 can be chosen in ways, similarly 2 white socks can be selected in 4 ways.

P(same colour)





Question 20.

Mark the correct alternative in each of the following:

If S is the sample space and and S = A∪B, where A and B are two mutually exclusive events, then P(A) =

A. 1/4

B. 1/2

C. 3/4

D. 3/8


Answer:

P(A) = 1/3 P(B) → Given


3 P(A) = P(B) → (1)


Now, since it’s given that A and B are mutually exclusive:


P(A ⋂ B) = 0 → (2)


Now then,


P(S) = P(A⋃B) = 1 → (3)


P(A⋃B) = P(A) + P(B) + P(A⋂B)


P(A) + P(B) = 1 → From (2) & (3)


P(A) + 3 P(A) = 1 → From (1)


4 P(A) = 1


P(A) = 1/4


Question 21.

Mark the correct alternative in each of the following:

If A and B are two events, then

A.

B. 1 – P (A) – P(B)

C. P(A) + P(B) – P (AB)

D.


Answer:


In the give picture we have,


= (1)


A⋂B = (2)


= (3)


Now, according to the addition theorem of probability we get the equation:




Question 22.

Mark the correct alternative in each of the following:

If P (A ∪ B) = 0.8 and P (A ∩ B) = 0.3, then

A. 0.3

B. 0.5

C. 0.7

D. 0.9


Answer:

For events A and B, addition theorem of probability →


P(A⋃B) = P(A) + P(B) – P(A⋂B) → (1)


Also,


→ (2)


→ (3)


Using equation (1)


0.8 = P(A) + P(B) – 0.3


P(A) + P(B) = 1.1


1 – P() + 1 – P() = 1.1 → From (2) & (3)


2 – {P() + P()} = 1.1


P() + P() = 2 – 1.1


P() + P() = 0.9


Question 23.

Mark the correct alternative in each of the following:

A bag X contains 2 white and 3 black balls and another bag Y contains 4 white and 2 black balls. One bag is selected at random and a ball is drawn from it. Then, the probability chosen to be white is

A. 2/15

B. 7/15

C. 8/15

D. 14/15


Answer:

Let A be the event of drawing a white ball.


Event M1 = Selecting bag X


Event M2 = Selecting bag Y


Let’s see what all probabilities we can find out from the given problem:


→ Selecting bag X = P(M1) =


→ Selecting bag Y = P(M2) =


→ Drawing white ball from bag X = P =


→ Drawing white ball from bag Y = P = =


Now, according to law of total probability →






Question 24.

Mark the correct alternative in each of the following:

Two persons A and B take turns in throwing a pair of dice. The first person to throw 9 from both dice will be awarded the prize. If A throws first, then the probability that B wins the game is

A. 9/17

B. 8/17

C. 8/9

D. 1/9


Answer:

Let’s find out the probability of getting ‘9’ from throw of a pair of dice first:


9 can be obtained in the given 4 cases →


[(3,6), (4,5), (5,4), (6,3)]


Getting 9 →


Not getting 9 →


B will win when A won’t get 9 in his/her throw and B will get 9 in his/her throw, i.e. B wins after 1 round of throws, P


Now then,


P(B wins) = P(Getting 9 in 2nd throw) + P(Getting 9 in 4th throw) + P(Getting 9 in 6th throw) + . . . . . . .



→ GP (Geometric Progression)





Question 25.

Mark the correct alternative in each of the following:

The probability that in a year of 22nd century chosen at random, there will be 53 Sundays, is

A. 3/28

B. 2/28

C. 7/28

D. 5/28


Answer:

P(53 Sundays) = P(Leap Years)P(53 Sundays in a Leap Year) + P(Non-Leap Years)P(53 Sundays in a Non-Leap Year) → (1)


Now,


P(53 Sundays in a Leap Year) → (52 Weeks + 2 days → Saturday + Sunday or Sunday + Monday)


P(53 Sundays in a non-Leap Year) → (52 Weeks + 1 day)


Next step in solving this problem requires the total number of leap years in 22nd Century.


Leap Years = 24


Non Leap Years = 76


P(Leap Year)


P(Non-Leap Year)


We now have all the required information, using equation (1) we get →






Question 26.

Mark the correct alternative in each of the following:

From a set of 100 cards numbered 1 to 100, one card is drawn at random. The probability that the number obtained on the card is divisible by 6 or 8 but not by 24 is

A. 6/25

B. 1/5

C. 1/6

D. 2/5


Answer:

No. divisible by 6 between 1-100 (A) = 16


No. divisible by 8 between 1-100 (B) = 12


No. divisible by 6 and 8 between 1-100 (C) = 4


No. divisible by 24 between 1-100 (D) = 4


P(No. divisible by 6 or 8) = P(A) + P(B) – P(C)





P(divisible by 6 or 8 but not by 24) = P(divisible by 6 or 8) – P(divisible by 24)





Question 27.

Mark the correct alternative in each of the following:

If A and B are two events such that then P(B/A) =

A. 1/10

B. 1/8

C. 7/8

D. 17/20


Answer:

P(A) = , P(A⋂B) = → (Given)


Now we know,






Question 28.

Mark the correct alternative in each of the following:

If A and B are two events associated to a random experiment such that and P(B) = 17/20, then P(A/B) =

A. 14/17

B. 17/20

C. 7/8

D. 1/8


Answer:


Now we know,






Question 29.

Mark the correct alternative in each of the following:

Associated to a random experiment two events A and B are such that P(B) = 3/5, P(A/B) = 1/2 and P(A ∪B) = 4/5. The value of P(A) is

A. 3/10

B. 1/2

C. 1/10

D. 3/5


Answer:

According to the addition theorem of probability for two events A and B we have,


P(A⋃B) = P(A) + P(B) – P(A⋂B)


P(A) + P(B) – P(A⋂B) = → (1)


Now,



→ (Given)


→ (Given)



Now using Equation (1),








Question 30.

Mark the correct alternative in each of the following:

If P(A) = 3/10, P(B) = 2/5 and P(A ∪B) = 3/5, then P(A/B) + P(B/A) equals

A. 1/4

B. 7/12

C. 5/12

D. 1/3


Answer:

P(A) = , P(B) = , P(A⋃B) = → (Given)


According to the addition theorem of probability for two events A and B we have,


P(A) + P(B) – P(A⋂B) = P(A⋃B)






Now then,







Question 31.

Mark the correct alternative in each of the following:

Let P(A) = 7/13, P(B) = 9/13 and P(A ∩ B) = 4/13. Then

A. 5/9

B. 4/9

C. 4/13

D. 6/13


Answer:


Now,


→ (1)



→ Refer to the Picture





Now using equation (1),






Question 32.

Mark the correct alternative in each of the following:

If and then is equal to

A. 5/6

B. 5/7

C. 25/42

D. 1


Answer:

→ (Given)


To find →


→ (1)


Let’s find the required values first and then we can put them into equation (1) to find the answer.


P(A) = 1 – P()


P() = 1 – P(A)




P(B) = 1 – P()


= 1 – P(B)




Also, according to the addition theorem of probability,


P(A⋃B) = P(A) + P(B) – P(A⋂B)






Also,



= 1 – P(A⋃B)




Now let’s put all the values in Equation (1)


P=






Question 33.

Mark the correct alternative in each of the following:

If A and B are two events such that then equals

A. 1/12

B. 3/4

C. 1/4

D. 3/16


Answer:

→ (Given)



= 1 – P(A⋃B) → (1)


Now,







Also, according to the addition theorem of probability,


P(A⋃B) = P(A) + P(B) – P(A⋂B)






Now using the value of P(A⋃B) in Equation (1) we get,


P() = 1 – P(A⋃B)




Question 34.

Mark the correct alternative in each of the following:

Let A and B be two events such that and Then P(A/B) is equals to

A. 2/5

B. 3/8

C. 3/20

D. 6/25


Answer:

→ (Given)


Now, according to the addition theorem of probability,


P(A⋃B) = P(A) + P(B) – P(A⋂B)


P(A) + P(B) – P(A⋂B)







According to the figure we get,






Now then,







Question 35.

Mark the correct alternative in each of the following:

If and then

A. 1/5

B. 4/5

C. 1/2

D. 1


Answer:

→ (Given)



= 1 – P(A⋃B) → (1)



= 1 – [P(A) – P (A⋂B)] → (2)


Now,




P(A⋂B) = P(B) 1/2


P(A⋂B)


→ (3)


According to the addition theorem of probability,


P(A⋃B) = P(A) + P(B) – P(A⋂B)


→ From (3) and (Given)




→ (4)


Now using Equation (1) and (2)


P (A⋃B) + P() = 1 – P(A⋃B) + 1 – [P(A) – P (A⋂B)]







Question 36.

Mark the correct alternative in each of the following:

If P(A) = 0.4, P(B) = 0.8 and P(B/A) = 0.6, then P(A ∪ B) =

A. 0.24

B. 0.3

C. 0.48

D. 0.96


Answer:

P(A) =0.4 , P =0.6 , P(B) = 0.8 → (Given)


P =0.6



P(A⋂B) = P(A) 0.6


P(A⋂B) = 0.4 0.6


P(A⋂B) = 0.24 → (1)


Now, according to the addition theorem of probability,


P(A⋃B) = P(A) + P(B) – P(A⋂B)


= 0.4 + 0.8 – 0.24 → From (1)


= 1.2 - 0.24


= 0.96


Question 37.

Mark the correct alternative in each of the following:

If and then

A. 1/5

B. 3/10

C. 1/2

D. 3/5


Answer:

P(B) = , P = , P(A⋃B) = → (Given)


Now as we know,




P(A⋂B) = P(B) 1/2


P(A⋂B)


→ (1)


Now, according to the addition theorem of probability,


P(A⋃B) = P(A) + P(B) – P(A⋂B)





→ (2)


Now according to the image:



P = P(B) – P(A⋂B)



→ (3)


Also, P( = 1 – P(A)



→ (4)


Now,






Question 38.

Mark the correct alternative in each of the following:

If A and B are two events such that P(A) = 0.4, P(B) = 0.3 and P(A ∪ B) = 0.5, then equals.

A. 2/3

B. 1/2

C. 3/10

D. 1/5


Answer:

P(A) =0.4 , P(B) =0.3 , P(A⋃B) = 0.5 → (Given)


Now, according to the addition theorem of probability,


P(A⋃B) = P(A) + P(B) – P(A⋂B)


0.5 = 0.4 + 0.3 – P(A∩B)


P(A∩B) = 0.7 – 0.5


P(A∩B) = 0.2 → (1)


Now according to the image:



P = P(A) – P(A⋂B)


= 0.4 – 0.2


= 0.2




Question 39.

Mark the correct alternative in each of the following:

If A and B are two events such that A ≠ ϕ, B = ϕ, then

A.

B. P(A/B) = P(A) P(B)

C. P(A/B) = P(B/A) = 1

D.


Answer:

According to Conditional Probability :


If A and B are two events associated with a random experiment such that A ϕ and B ϕ then,



So, option (A) is correct.


Question 40.

Mark the correct alternative in each of the following:

If A and B are two events such that P(A) ≠ 0 and P(B) ≠ 1, then

A. 1 – P(A/B)

B.

C.

D.


Answer:

→ (1)


Also,



= 1 – P(A⋃B) → (2)


Using (2) in Equation (1) we get,



Question 41.

Mark the correct alternative in each of the following:

If the events A and B are independent, then P(A ∩ B) is equal to

A. P(A) + P(B)

B. P(A) – P(B)

C. P(A) P(B)

D.


Answer:

As we know,


→ (1)


Now as events A and B are independent events,


→ (2)


Using the value obtained in (2) in (1) we get,



P(A∩B) = P(B) P


P(A∩B) = P(B) P(A)


Question 42.

Mark the correct alternative in each of the following:

If A and B are two independent events with then equals.

A. 4/15

B. 8/45

C. 1/3

D. 2/9


Answer:

→ (Given)


Now as we know,



= 1 – P(A⋃B) → (1)


Since, A and B are two independent events,


P(A∩B) = P(A) P(B)




→ (2)


Now, according to the addition theorem of probability,


P(A⋃B) = P(A) + P(B) – P(A⋂B)


→ From (2)




→ (3)


Using (3) in equation (1) we get,



= 1 – P(A⋃B)




Question 43.

Mark the correct alternative in each of the following:

If A and B are two independent events such that P(A) = 0.3, P(A ∪ B) = 0.5, then P(A/B) – P(B/A) =

A. 2/7

B. 3/35

C. 1/70

D. 1/7


Answer:

P(A) = 0.3 , P(A∪B) = 0.5 → (Given)


Since, A and B are two independent events,


P(A∩B) = P(A) P(B)


P(A∩B) = 0.3 P(B) → (1)


Also, according to the addition theorem of probability,


P(A⋃B) = P(A) + P(B) – P(A⋂B)


0.5 = 0.3 + P(B) – 0.3P(B) → From (Given) & (1)


0.7P(B) = 0.2


P(B) = = → (2)


Putting value of P(B) in equation (1) we get,


P(A∩B) = 0.3 =


P(A∩B) = → (3)


Now,



→ From (3) & (2) and (Given)





Question 44.

Mark the correct alternative in each of the following:

A flash light has 8 batteries out of which 3 are dead. If two batteries are selected without replacement and tested, the probability that both are dead is

A. 3/28

B. 1/14

C. 9/64

D. 33/56


Answer:

Total Number of Batteries = 8


Total Number of Dead Batteries = 3


Now Let,


M = Event of selecting the first dead battery



N= Event of selecting the second dead battery


→ (Without replacement)


P(Both dead batteries are selected) = P(M) P(N)




Question 45.

Mark the correct alternative in each of the following:

A bag contains 5 red and 3 blue balls. If 3 balls are selected drawn at random without replacement, then the probability of getting exactly one red ball is

A. 15/29

B. 15/56

C. 45/196

D. 135/392


Answer:

Total Red Balls = 5


Total Blue Balls = 3


M = Event getting Red ball



N = Event getting Blue ball



Now,


P(Getting Exactly one red ball) = P(RBB) + P(BRB) + P(BBR)





Question 46.

Mark the correct alternative in each of the following:

A bag contains 5 red and 3 blue balls. If 3 balls are drawn at random without replacement, then the probability that exactly two of the three balls were red, the first ball being red, is

C. 15/28 D. 5/28


Answer:

Total Red Balls = 5


Total Blue Balls = 3


M = Event getting Red ball


N = Event getting Blue ball


Now,


P(Getting exactly 2 red balls of 3 balls, the first ball being red) →


→ (Since 1st ball is already red, P(red ball) = and so on..)




Question 47.

Mark the correct alternative in each of the following:

In a college 30% students fail in Physics, 25% fail in Mathematics and 10% fail in both. One student is chosen at random. The probability that she fails in Physics if she has failed in Mathematics is

A. 1/10

B. 1/3

C. 2/5

D. 9/20


Answer:

A = Failing in Physics


B = Failing in Mathematics


P(A) = 30%




P(B) = 25%



→ (Given)


P(A∩B) = 10%




Now we know,







Question 48.

Mark the correct alternative in each of the following:

Three person, A, B and C fire a target in turn starting with A. Their probabilities of hitting the target are 0.4, 0.3 and 0.2 respectively. The probability of two hits is

A. 0.024

B. 0.452

C. 0.336

D. 0.188


Answer:

A = Event of Person A hitting the target.


P(A) = 0.4


P = 1 – P(A) = 1 – 0.4


P = 0.6 → (1)


B = Event of Person B hitting the target.


P(B) = 0.3


P = 1 – P(B) = 1 – 0.3


P = 0.7 → (2)


C = Event of Person C hitting the target.


P(C) = 0.2


P = 1 – P(C) = 1 – 0.2


P = 0.8 → (3)


Now then,



= 0.4 × 0.3 × 0.8 + 0.4 × 0.7 × 0.2 + 0.6 × 0.3 × 0.2


= 0.096 + 0.056 + 0.036


= 0.188


Question 49.

Mark the correct alternative in each of the following:

A and B are two students. Their chances of solving a problem correctly are 1/3 and 1/4 respectively. If the probability of their making common error is 1/20 and they obtain the same answer, then the probability of their answer to be current is

A. 10/13

B. 13/120

C. 1/40

D. 1/12


Answer:

E1 = Both Solve correctly




E2 = Both Solve incorrectly




A = They get the same result


Using Baye’s Theorem →






Question 50.

Mark the correct alternative in each of the following:

Two cards are drawn from a well shuffled deck of 52 playing cards with replacement. The probability that both cards are queen is

A.

B.

C.

D.


Answer:

M = Event that Queen is draw in in first draw.


P(M) =


N = Event that Queen is drawn in second draw.


P(N) =


Now, Probability that both the cards drawn are queen:


P = P(M) P(N)




Question 51.

Mark the correct alternative in each of the following:

A box contains 3 orange balls, 3 green balls and 2 blue balls. Three balls are drawn at random from the box without replacement. The probability of drawing 2 green balls and one blue balls is

A. 167/168

B. 1/28

C. 2/21

D. 3/28


Answer:

P(G) = Probability of drawing a green ball =


P(B) = Probability of drawing a blue ball =


P(O) = Probability of drawing an orange ball =


Important → When taking the probability of green/ball in 2nd draw, remember to reduce the total number of balls and number of green/blue ball


While finding the probability. Example – P(GGB) =


Now Probability of drawing 2 green balls and one blue ball :


P = P(GGB) + P(GBG) + P(BGG)





Question 52.

Mark the correct alternative in each of the following:

If two events are independent, then

A. they must be mutually exclusive

B. the sum of their probabilities must be equal to 1

C. (a) and (b) both are correct

D. none of the above is correct


Answer:

Let A and B be two independent events, then,


P(A∩B) = P(A) P(B)


P(A∩B) 0 → Not mutually exclusive


Or,


P(A) + P(B) 1 → Their sum of probabilities is not 1


None of the above is correct.


Question 53.

Mark the correct alternative in each of the following:

Two dice are thrown. If it is known that the sum of the numbers on the dice was less than 6, the probability of getting a sum 3, is

A. 1/18

B. 5/18

C. 1/5

D. 2/5


Answer:

A = Event of getting a sum of 3.


A = { (1,2) , (2,1) }


B = Event of sum of the numbers less than 6.


B = { (1,1) , (1,2) , (1,3) , (1,4) , (2,1) , (2,2) , (2,3) , (3,1) , (3,2) , (4,1) }


(A∩B) = { (1,2) , (2,1) } → (1)


Now, we have →


n(A) = 2


n(B) = 10


n(A∩B) = n(A) = 2 → (2)


Now,





Question 54.

Mark the correct alternative in each of the following:

If A and B are such that and then

A. 9/10

B. 10/9

C. 8/9

D. 9/8


Answer:

P(A∪B) = , P = → (Given)


Now,


P = P(A∩B) → (1)


P(A∩B) = 1 - P(A∩B)


→ From (Given) & (1)



Also,


P = 1 – P(A) → (2)


P = 1 – P(B) → (3)


Now, according to the addition theorem of probability:


P(A⋃B) = P(A) + P(B) – P(A⋂B)




→ From (2) & (3)




Question 55.

Mark the correct alternative in each of the following:

If A and B are two events such that P(A/B) = p, P(A) = p, P(B) = 1/3 and then p =

A. 2/3

B. 3/5

C. 1/3

D. 3/4


Answer:

P = p , P(A) = p , P(B) = , P(A∪B) = → (Given)


Now,



→ From (Given)


P(A∩B) = → (1)


Now, according to the addition theorem of probability:


P(A⋃B) = P(A) + P(B) – P(A⋂B)







Question 56.

Mark the correct alternative in each of the following:

A die is thrown and a card is selected at random from a deck of 52 playing cards. The probability of getting an even number of the die and a spade card is

A. 1/2

B. 1/4

C. 1/8

D. 3/4


Answer:

M = Event of getting an even number.


P(M) = 1/2 → (1)


N = Event of getting a spade card.


P(N) = 1/4 → (2)


P(Even number of the die and spade card) = P(MN)


→ From (1) & (2)



Question 57.

Mark the correct alternative in each of the following:

Assume that in a family, each child is equally likely to be a boy or a girl. A family with three children is chosen at random. The probability that the eldest child is a girl given that the family ahs at least one girl is

A. 1/2

B. 1/3

C. 2/3

D. 4/7


Answer:

The sample set we have, where the first letter represents the eldest child of the family →


S = { GGG, GBG, GGB, GBB , BBB, BBG, BGB, BGG }


Now let,


M = Event of choosing a family with a girl as the eldest child.


= { GGG, GBG, GGB, GBB }


N = Event of choosing a family with at-least one girl child.


= { GGG, GBG, GGB, GBB, BBG, BGB, BGG }


We have,


n(M) = 4


n(N) = 7


n(M∩N) = 4


Now,




Question 58.

Mark the correct alternative in each of the following:

Let A and B be two events. If P(A) = 0.2, P(B) = 0.4, P(A ∪ B) = 0.6, then P(A/B) is equal to

A. 0.8

B. 0.5

C. 0.3

D. 0


Answer:

P(A) = 0.2 , P(B) = 0.4 , P(A∪B) = 0.6 → (Given)


According to the addition theorem of probability,


P(A⋃B) = P(A) + P(B) – P(A⋂B)


0.6 = 0.2 + 0.4 – P(A∩B)


0.6 = 0.6 – P(A∩B)


P(A∩B) = 0 → (1)


Now,




= 0


Question 59.

Mark the correct alternative in each of the following:

Let A and B be two events such that P(A) = 0.6, P(B) = 0.2 and P(A/B) = 0.5. Then equals.

A. 1/10

B. 3/10

C. 3/8

D. 6/7


Answer:

P(A) = 0.6 , P(B) = 0.2 , P = 0.5 → (Given)



→ (1)



= 1 – 0.2


= 0.8


Also,



= 1 – P(A⋃B) → (3)


Using (2) & (3) in Equation (1) we get,


→ (3)


Now,



P(A∩B) = 0.5 P(B) → From (Given)


P(A∩B) = 0.5 0.2


P(A∩B) = 0.1


According to the addition theorem of probability,


P(A⋃B) = P(A) + P(B) – P(A⋂B)


= 0.6 + 0.2 – 0.1


= 0.8 – 0.1


= 0.7 → (4)


Putting (4) in equation (3) we get,